Guyton and Hall Physiology 01

 




























































































































































Review

FOURTH EDITION

John E. Hall, PhD

Arthur C. Guyton Professor and Chair, Department of Physiology and Biophysics, Director of the

Mississippi Center for Obesity Research, University of Mississippi Medical Center, Jackson,

Mississippi


Table of Contents

Cover image

Title page

Copyright

Contributors

Preface

Unit I. Introduction to Physiology: The Cell and General Physiology

Unit II. Membrane Physiology, Nerve, and Muscle

Unit III. The Heart

Unit IV. The Circulation

Unit V. The Body Fluids and Kidneys

Unit VI. Blood Cells, Immunity, and Blood Coagulation

Unit VII. Respiration

Unit VIII. Aviation, Space, and Deep-Sea Diving Physiology

Unit IX. The Nervous System: A. General Principles and Sensory Physiology

Unit X. The Nervous System: B. The Special Senses

Unit XI. The Nervous System: C. Motor and Integrative Neurophysiology

Unit XII. Gastrointestinal Physiology

Unit XIII. Metabolism and Temperature Regulation

Unit XIV. Endocrinology and Reproduction

Unit XV. Sports Physiology

Normal Values for Selected Common Laboratory Measurements


Copyright

Elsevier

3251 Riverport Lane

St. Louis, Missouri 63043

GUYTON AND HALL PHYSIOLOGY REVIEW, FOURTH EDITION ISBN: 978-0-323-

63999-6

Copyright © 2021 by Elsevier, Inc. All rights reserved.

No part of this publication may be reproduced or transmied in any form or by any means,

electronic or mechanical, including photocopying, recording, or any information storage

and retrieval system, without permission in writing from the publisher. Details on how to

seek permission, further information about the Publisher’s permissions policies and our

arrangements with organizations such as the Copyright Clearance Center and the

Copyright Licensing Agency, can be found at our website: www.elsevier.com/permissions.

This book and the individual contributions contained in it are protected under copyright by

the Publisher (other than as may be noted herein).

Notice

Practitioners and researchers must always rely on their own experience and knowledge in

evaluating and using any information, methods, compounds or experiments described

herein. Because of rapid advances in the medical sciences, in particular, independent

verification of diagnoses and drug dosages should be made. To the fullest extent of the law,

no responsibility is assumed by Elsevier, authors, editors or contributors for any injury

and/or damage to persons or property as a maer of products liability, negligence or

otherwise, or from any use or operation of any methods, products, instructions, or ideas

contained in the material herein.

Previous editions copyrighted 2016, 2011, 2006.

Library of Congress Control Number: 2020942162

Publisher: Elyse O’Grady

Director, Content Development: Ellen M. Wurm-Cuer

Senior Content Development Specialist: Kathleen Nahm

Publishing Services Manager: Shereen Jameel

Project Manager: Manikandan Chandrasekaran

Cover Design and Design Direction: Margaret Reid

Printed in Canada

Last digit is the print number: 9 8 7 6 5 4 3 2 1


Contributors

Thomas H. Adair, PhD , Professor of Physiology and Biophysics, University of

Mississippi Medical Center, Jackson, Mississippi, Units II, IX, X, XI, XII, and XIII

Alejandro R. Chade, MD , Professor of Physiology and Biophysics, University of

Mississippi Medical Center, Jackson, Mississippi, Units III and IV

Joey P. Granger, PhD , Dean, School of Graduate Studies in theHealth Sciences,

Professor of Physiology and Biophysics, University of Mississippi Medical Center, Jackson,

Mississippi, Unit IV

John E. Hall, PhD , Professor and Chair of Physiology and Biophysics, University of

Mississippi Medical Center, Jackson, Mississippi, Units I, V, and XIII

Robert L. Hester, PhD , Professor of Physiology and Biophysics, University of

Mississippi Medical Center, Jackson, Mississippi, Units VII, VIII, and XV

Michael J. Ryan, PhD , Professor of Physiology and Biophysics, University of

Mississippi Medical Center, Jackson, Mississippi, Unit XIV

James G. Wilson, MD , Professor Emeritus of Physiology and Biophysics, University of

Mississippi Medical Center, Jackson, Mississippi, Unit VI


Preface

The main purpose of this book is to provide students a tool for assessing their mastery of

physiology as presented in the Guyton and Hall Textbook of Medical Physiology, 14th edition.

Self-assessment is an important component of effective learning, especially when

studying a subject as complex as medical physiology. Guyton & Hall Physiology Review is

designed to provide a comprehensive review of medical physiology through multiplechoice questions and explanations of the answers. Medical students preparing for the

United States Medical Licensure Examinations (USMLE) will also find this book useful

because many of the test questions have been constructed according to the USMLE format.

The questions and answers in this review are based on Guyton and Hall Textbook of Medical

Physiology, 14th edition (TMP 14). More than 1000 questions and answers are provided, and

each answer is referenced to the Textbook of Medical Physiology to facilitate a more complete

understanding of the topic. Illustrations are used to reinforce basic concepts. Some of the

questions incorporate information from multiple chapters to test your ability to apply and

integrate the principles necessary for mastery of medical physiology.

An effective way to use this book is to allow an average of 1 minute for each question in a

unit, approximating the time limit for a question in the USMLE examination. As you

proceed, indicate your answer next to each question. After finishing the questions and

answers, verify your answers and carefully read the explanations provided. Read the

additional material referred to in the Textbook of Medical Physiology, especially when

incorrect answers were chosen.

Guyton and Hall Physiology Review should not be used as a substitute for the

comprehensive information contained in the Textbook of Medical Physiology. Its main purpose

is to assess your knowledge of physiology gained from study of the Textbook of Medical

Physiology and other sources and to strengthen your ability to apply and integrate this

knowledge.

We have aempted to make this review as accurate as possible, and we hope that it will

be a valuable tool for your study of physiology. We invite you to send us your critiques,

suggestions for improvement, and notifications of any errors.

I am grateful to each of the contributors for their careful work on this book. I also wish to

express my thanks to Kathleen Nahm, Manikandan Chandrasekaran, Jennifer Schreiner,

Rebecca Gruliow, Elyse O’Grady, and the rest of the Elsevier staff for their editorial and

production excellence.

John E. Hall


Unit I: Introduction to Physiology

The Cell and General Physiology

1. If the feedback gain of a control system is -3.0, this means that the

system is

A) A negative feedback system capable of correcting 1/3 of the

initial disturbance to the system

B) A negative feedback system capable of correcting 2/3 of the

initial disturbance to the system

C) A negative feedback system capable of correcting 3/4 of the

initial disturbance to the system

D) A positive feedback system capable of correcting 1/3 of the

initial disturbance to the system

2. Most cells, except for fat cells, are composed mainly of

A) Proteins

B) Ions

C) Water

D) Microfilaments/cell cytoskeleton

E) Secretory vesicles

3. Organelles that neutralize drugs and toxins are

A) Nuclei

B) Mitochondria

C) Lysosomes

D) Peroxisomes

E) Endoplasmic reticulum

4. The most abundant cell membrane lipids are

A) Sphingolipids

B) Phospholipids

C) Cholesterol

D) Triglycerides

E) Sterols

5. The first stage of mitosis is called

A) Anaphase

B) Prophase

C) Prometaphase

D) Metaphase

E) Telophase

6. The region of repetitive nucleotide sequences located at each end of

a chromatid is called

A) Okazaki fragment

B) Replication fork

C) Telomere

D) Centriole

E) Lagging strand

7. Which of the following statements about DNA replication is

incorrect?

A) Both strands of the DNA in each chromosome are replicated

B) Both strands of the DNA helix are replicated in small portions

and then assembled, similar to the transcription of RNA

C) Before DNA can be replicated, the double stranded molecule

must be “unzipped” into two single strands

D) Uncoiling of DNA helixes is achieved by DNA helicase

enzymes

E) Once the DNA strands have been separated, a short piece of

RNA called an RNA primer binds to the 3’ end of the leading

strand

8. Which of the following statements about cell differentiation is

correct?

A) Differentiation results from selective loss of different genes

from cells

B) Differentiation results from selective repression of different

gene promoters

C) Differentiation results from selective activation of telomerase

in different cells

D) Differentiation results mainly from mutations of genes

9. Which statement about microRNAs (miRNAs) is correct?

A) miRNAs are formed in the cytoplasm and repress translation

or promote degradation of messenger RNA (mRNA) before it

can be translated

B) miRNAs are formed in the nucleus and then processed in the

cytoplasm by the dicer enzyme

C) miRNAs are short (21 to 23 nucleotide) double-stranded RNA

fragments that regulate gene expression

g g g p

D) miRNAs repress gene transcription

10. Compared with the intracellular fluid, the extracellular fluid has

__________ sodium ion concentration, __________ potassium ion

concentration, __________ chloride ion concentration, and

__________ phosphate ion concentration.

A) Lower, lower, lower, lower

B) Lower, higher, lower, lower

C) Lower, higher, higher, lower

D) Higher, lower, higher, lower

E) Higher, higher, lower, higher

F) Higher, higher, higher, higher

11. Which of the following events does not occur during the process of

mitosis?

A) Condensation of the chromosomes

B) Replication of the genome

C) Fragmentation of the nuclear envelope

D) Alignment of the chromatids along the equatorial plate

E) Separation of the chromatids into two sets of 46 “daughter”

chromosomes

12. The term “glycocalyx” refers to what?

A) The negatively charged carbohydrate chains that protrude

into the cytosol from glycolipids and integral glycoproteins

B) The negatively charged carbohydrate layer on the outer cell

surface

C) The layer of anions aligned on the cytosolic surface of the

plasma membrane

D) The large glycogen stores found in “fast” muscles

E) A mechanism of cell–cell aachment

13. Which statement is incorrect?

A) The term “homeostasis” describes the maintenance of nearly

constant conditions in the body

B) In most diseases, homeostatic mechanisms are no longer

operating in the body

C) The body’s compensatory mechanisms often lead to

deviations from the normal range in some of the body’s

functions

D) Disease is generally considered to be a state of disrupted

homeostasis

Questions 14–16

A) Nucleolus

B) Nucleus

C) Agranular endoplasmic reticulum

D) Granular endoplasmic reticulum

E) Golgi apparatus

F) Endosomes

G) Peroxisomes

H) Lysosomes

I) Cytosol

J) Cytoskeleton

K) Glycocalyx

L) Microtubules

For each of the scenarios described below, identify the most likely

subcellular site listed above for the deficient or mutant protein.

14. The abnormal cleavage of mannose residues during the posttranslational processing of glycoproteins results in the

development of a lupus-like autoimmune disease in mice. The

abnormal cleavage is due to a mutation of the enzyme αmannosidase II.

15. The observation that abnormal cleavage of mannose residues from

glycoproteins causes an autoimmune disease in mice supports the

role of this structure in the normal immune response.

16. Studies completed on a 5-year-old boy show an accumulation of

cholesteryl esters and triglycerides in his liver, spleen, and

intestines and calcification of both adrenal glands. Additional

studies indicate the cause to be a deficiency in acid lipase A

activity.

Questions 17–20

A) Nucleolus

B) Nucleus

C) Agranular endoplasmic reticulum

D) Granular endoplasmic reticulum

E) Golgi apparatus

F) Endosomes

G) Peroxisomes

H) Lysosomes

I) Cytosol

J) Cytoskeleton

K) Glycocalyx

L) Microtubules

Match the cellular location for each of the steps involved in the synthesis

and packaging of a secreted protein listed below with the correct term

from the list above.

17. Protein condensation and packaging

18. Initiation of translation

19. Gene transcription

20. Worn-out organelles are transferred to lysosomes by which of the

following?

A) Autophagosomes

B) Granular endoplasmic reticulum

C) Agranular endoplasmic reticulum

D) Golgi apparatus

E) Mitochondria

21. Which of the following does not play a direct role in the process of

transcription?

A) Helicase

B) RNA polymerase

C) Chain-terminating sequence

D) “Activated” RNA molecules

E) Promoter sequence

22. Which statement is true for both pinocytosis and phagocytosis?

A) Involves the recruitment of actin filaments

B) Occurs spontaneously and nonselectively

C) Endocytotic vesicles fuse with ribosomes that release

hydrolases into the vesicles

D) Is only observed in macrophages and neutrophils

E) Does not require ATP

23. Which statement is incorrect?

A) Proto-oncogenes are normal genes that code for proteins that

control cell growth

B) Proto-oncogenes are normal genes that code for proteins that

control cell division

C) Inactivation of anti-oncogenes protects against the

development of cancer

D) Several different simultaneously activated oncogenes are

often required to cause cancer

24. Which statement about feedback control systems is incorrect?

A) Most control systems of the body act by negative feedback

B) Positive feedback usually promotes stability in a system

C) Generation of nerve actions potentials involves positive

feedback

D) Feed-forward control is important in regulating muscle

activity

25. Which of the following cell organelles is responsible for producing

adenosine triphosphate (ATP), the energy currency of the cell?

A) Endoplasmic reticulum

B) Mitochondria

C) Lysosomes

D) Golgi apparatus

E) Peroxisomes

F) Ribosomes

26. Which statement about mRNA is correct?

A) mRNA carries the genetic code to the cytoplasm

B) mRNA carries activated amino acids to the ribosomes

C) mRNA is composed of single-stranded RNA molecules of 21

to 23 nucleotides that can regulate gene transcription

D) mRNA forms ribosomes

27. “Redundancy” or “degeneration” of the genetic code occurs

during which step of protein synthesis?

A) DNA replication

B) Transcription

C) Post-transcriptional modification

D) Translation

E) Protein glycosylation

Answers

1. C) The feedback gain of a control system is calculated as the amount of

correction divided by the remaining error of the system. A feedback

gain of -3.0 means that 3/4 of the initial error was corrected by the

system. For example, if the initial error was 4 units and 1 unit of error

remains after correction, then the amount of correction is -3 (from 4 to

1), the remaining error is 1, and the feedback gain is -3.0.

TMP14 pp. 8–9

2. C) Most cells, except for fat cells, are composed mainly of water in a

concentration of 70% to 85 %. After water, the most abundant substances

in most cells are proteins, which normally constitute 10% to 20 % of the

cell mass.

TMP14 p. 13

3. D) Peroxisomes contain oxidases capable of combining oxygen with

hydrogen ions derived from different intracellular chemicals to form

hydrogen peroxide (H2O2

), a highly oxidizing substance used in

association with catalase, another oxidase enzyme present in large

quantities in peroxisomes. These enzymes oxidize and neutralize many

drugs and toxins that might otherwise be poisonous to the cell.

TMP14 p. 18

4. B) The basic cell membrane lipid bilayer is composed of proteins and

three main types of lipids: phospholipids, sphingolipids, and

cholesterol. The approximate composition is 55% proteins, 25%

phospholipids, 13% cholesterol, 4% other lipids, and 3% carbohydrates.

TMP14 pp. 15–16

5. B) The first stage of mitosis, the process by which the cell splits into two

new cells, is called prophase (see figure below).

TMP14 pp. 41–43

Stages of cell reproduction. A, B, and C, Prophase. D, Prometaphase. E,

Metaphase. F, Anaphase. G and H, Telophase

6. C) Telomeres are repetitive nucleotide sequences located at each end of a

chromatid and serve as protective caps that prevent the chromosome

from deterioration during cell division. Without telomeres, the genomes

would progressively lose information and be truncated after each cell

division.

TMP14 p. 44

7. B) Both entire strands of the DNA helix in each chromosome are

replicated from end to end, rather than small portions of them, as occurs

in the transcription of RNA (see figure below).

The helical double-stranded structure of the gene. The outside strands are

composed of phosphoric acid and the sugar deoxyribose. The inner

molecules connecting the two strands of the helix are purine and pyrimidine

bases, which determine the “code” of the gene.

TMP14 pp. 41–42

8. B) Cell differentiation refers to changes in physical and functional

properties of cells as they proliferate in the embryo to form different

bodily structures and organs and results not from loss of genes but from

selective repression of different gene promoters.

TMP14 p. 45

9. A) The miRNAs are formed in the cytoplasm from pre-miRNAs and

processed by the enzyme dicer that ultimately assembles RNA-induced

silencing complex, which then generates miRNAs. The miRNAs

regulate gene expression by binding to the complementary region of the

RNA and repressing translation or promoting degradation of messenger

RNA before it can be translated by the ribosome.

TMP14 pp. 36–37

10. D) The extracellular fluid has relatively high concentrations of sodium

and chloride ions but lower concentrations of potassium and phosphate

than the intracellular fluid.

TMP14 p. 4

11. B) DNA replication occurs during the S phase of the cell cycle and

precedes mitosis. Condensation of the chromosomes occurs during the

prophase of mitosis. Fragmentation of the nuclear envelope occurs

during the prometaphase of mitosis. The chromatids align at the

equatorial plate during metaphase and separate into two complete sets

of daughter chromosomes during anaphase.

TMP14 p. 43

12. B) The cell “glycocalyx” is the loose negatively charged carbohydrate

coat on the outside of the surface of the cell membrane. The membrane

carbohydrates usually occur in combination with proteins or lipids in

the form of glycoproteins or glycolipids, and the “glyco” portion of

these molecules almost invariably protrudes to the outside of the cell.

TMP14 p. 16

13. B) The term homeostasis describes the maintenance of nearly constant

conditions in the internal environment of the body, and diseases are

generally considered to be states of disrupted homeostasis. However,

even in diseases, homeostatic compensatory mechanisms continue to

operate in an aempt sustain body functions at levels that permit life to

continue. These compensations may result in deviations from the

normal level of some body functions as a “trade-off” that is necessary to

maintain vital functions of the body.

TMP14 p. 4

14. E) Membrane proteins are glycosylated during their synthesis in the

lumen of the rough endoplasmic reticulum. Most post-translational

modification of the oligosaccharide chains, however, occurs dur ing the

transport of the protein through the layers of the Golgi apparatus

matrix, where enzymes such as α-mannosidase II are localized.

TMP14 pp. 16–17

15. K) The oligosaccharide chains that are added to glycoproteins on the

luminal side of the rough endoplasmic reticulum, and subsequently

modified during their transport through the Golgi apparatus, are

aached to the extracellular surface of the cell. This negatively charged

layer of carbohydrate moieties is collectively called the glycocalyx. It

participates in cell–cell interactions, cell–ligand interactions, and the

immune response.

TMP14 p. 16

16. H) Acid lipases, along with other acid hydrolases, are localized to

lysosomes. Fusion of endocytotic and autolytic vesicles with lysosomes

initiates the intracellular process that allows cells to digest cellular

debris and particles ingested from the extracellular milieu, including

bacteria. In the normal acidic environment of the lysosome, acid lipases

use hydrogen to convert lipids into fay acids and glycerol. Other acid

lipases include a variety of nucleases, proteases, and polysaccharidehydrolyzing enzymes.

TMP14 pp. 17–18

17. E) Secreted proteins are condensed, sorted, and packaged into secretory

vesicles in the terminal portions of the Golgi apparatus, also known as

the trans-Golgi network. It is here that proteins destined for secretion

are separated from those destined for intracellular compartments or

cellular membranes.

TMP14 p. 17

18. I) Initiation of translation, whether of a cytosolic protein, a membranebound protein, or a secreted protein, occurs in the cytosol and involves a

common pool of ribosomes. Only after the appearance of the Nterminus of the polypeptide is it identified as a protein destined for

secretion. At this point, the ribosome aaches to the cytosolic surface of

the rough endoplasmic reticulum. Translation continues, and the new

polypeptide is extruded into the matrix of the endoplasmic reticulum.

TMP14 pp. 37–38

19. B) All transcription events occur in the nucleus, regardless of the final

destination of the protein product. The resulting messenger RNA

molecule is transported through the nuclear pores in the nuclear

membrane and translated into either the cytosol or the lumen of the

rough endoplasmic reticulum.

TMP14 pp. 33–34

20. A) Autophagy is a housekeeping process by which obsolete organelles

and large protein aggregates are degraded and recycled (see figure at

right). Worn-out cell organelles are transferred to lysosomes by double

membrane structures called autophagosomes that are formed in the

cytosol.

TMP14 pp. 22–23

Schematic diagram of autophagy steps

21. A) Helicase is one of the many proteins involved in the process of DNA

replication. It does not play a role in transcription. RNA polymerase

binds to the promoter sequence and facilitates the addition of

“activated” RNA molecules to the growing RNA molecule until the

polymerase reaches the chain-terminating sequence on the template

DNA molecule.

TMP14 pp. 33–34, 42

22. A) Both pinocytosis and phagocytosis involve movement of the plasma

membrane. Pinocytosis involves invagination of the cell membrane,

whereas phago cytosis involves evagination. Both events require the

recruitment of actin and other cytoskeleton elements. Phagocytosis is

not spontaneous and is selective, being triggered by specific receptorligand interactions.

TMP14 pp. 21–22

23. C) Inactivation of anti-oncogenes, also called tumor suppressor genes,

can allow activation of oncogenes that lead to cancer. All the other

statements are correct.

TMP14 pp. 46–47

24. B) Positive feedback in a system generally promotes instability, rather

than stability, and in some cases even death. For this reason, positive

feedback is often called a “vicious cycle.” However, in some instances,

positive feedback can be useful. One example is the nerve action

potential where stimulation of the nerve membrane causes a slight

leakage of sodium that causes more opening of sodium channels, more

change of potential, and more opening of channels until an explosion of

sodium entering the interior of the nerve fiber creates the action

potential. Feed-forward control is used to apprise the brain whether a

muscle movement is performed correctly. If not, the brain corrects the

feed-forward signals that it sends to the muscles the next time the

movement is required. This mechanism is often called adaptive control.

TMP14 pp. 8–10

25. B) Mitochondria are often called the “powerhouses” of the cell and

contain oxidative enzymes that permit oxidation of the nutrients,

thereby forming carbon dioxide and water and at the same time

releasing energy. The liberated energy is used to synthesize “highenergy” ATP.

TMP14 pp. 18–19

26. A) mRNA molecules are long, single RNA strands that are suspended

in the cytoplasm and are composed of several hundred to several

thousand RNA nucleotides in unpaired strands. The mRNA carries the

genetic code to the cytoplasm for controlling the type of protein formed.

The transfer RNA transports activated amino acids to the ribosomes.

Ribosomal RNA, along with about 75 different proteins, forms ribosomes.

MicroRNAs are single-stranded RNA molecules of 21 to 23 nucleotides

that regulate gene transcription and translation.

TMP14 p. 35

27. D) During both replication and transcription, the new nucleic acid

molecule is an exact complement of the parent DNA molecule as a result

of predictable, specific, one-to-one base pairing. During the process of

translation, however, each amino acid in the new polypeptide is

encoded by a codon—a series of three consecutive nucleotides. Whereas

each codon encodes a specific amino acid, most amino acids can be

encoded for by multiple codons. Redundancy results because 60 codons

encode a mere 20 amino acids.

TMP14 pp. 34–36


Unit II: Membrane Physiology, Nerve,

and Muscle

1. A patch clamp experiment shows a single sodium ion channel that

opens and closes repeatedly causing the electrical current through the

channel to change from one value to another. The open time of the

sodium channel averages 0.4 milliseconds in this experiment. Which of

the following best describes the electrical current of this sodium

channel during the open and closed states (in picoamperes)?

A) Open: 3.2; closed: 3.3

B) Open: 0.4; closed: 0.4

C) Open: 0.4; closed: 3.2

D) Open: 3.1; closed: 0.4

E) Open: 0.4; closed: 2.0

F) Open: 0.4; closed: 0.6

2. Which of the following best describes the osmolarity of a solution

containing 150 millimolar NaCl, assuming a temperature of 37°C and a

dissociation constant of 0.93 (in milliosmoles)?

A) 150

B) 279

C) 300

D) 322

E) 393

3. A 64-year-old man has serum potassium of 2.8 mEq/l (reference range,

3.5–5.0 mEq/l). Which of the following sets of changes best describe the

resting membrane potential (Vm) and K+

 Equilibrium potential (EK) in

a typical neuron in this man compared with normal? (Assume normal

intracellular concentration of K +

.)

A) EK, less negative; Vm, less negative

B) EK, less negative; Vm, no change

C) EK, less negative; Vm, more negative

D) EK, more negative; Vm, less negative

E) EK, more negative; Vm, more negative

F) EK, more negative; Vm, no change

G) EK, no change; Vm, less negative

H) EK, no change; Vm, more negative

I) EK, no change; Vm, no change

Questions 4 and 5

In the figure shown, two compartments (Y and Z) are separated by an

artificial lipid bilayer without protein transporters. The relative concentrations

of test substances in compartments Y and Z at time zero are shown. Different

water volumes in compartments Y and Z are shown as diagrams A to E. Use

this information to answer the next two questions.

4. Which of the diagrams best represent the volumes of compartments Y

and Z at equilibrium when the test substance is NaCl?

A) A

B) B

C) C

D) D

E) E

5. Which of the diagrams best represent the volumes of compartments Y

and Z at equilibrium when the test substance is urea?

A) A

B) B

C) C

D) D

E) E

6. A model cell with three different transporters (X, Y, and Z) and a resting

membrane potential of −90 millivolts is shown. Consider the

intracellular and extracellular concentrations of all three ions to be

typical of a normal cell. Inhibition of transporter Y with ouabain is most

likely to cause which of the following changes in the intracellular

concentrations of sodium and calcium ions?

A) Decreased sodium; decreased calcium

B) Decreased sodium; increased calcium

C) Increased sodium; decreased calcium

D) Increased sodium; increased calcium

7. In the diagram shown, Em represents the measured initial membrane

potential for a hypothetical cell in vivo. In relation to this membrane

potential, the equilibrium potentials of three ions (X−

, Y−

, Z+) are

represented. Pick the path most likely taken by the membrane potential

when membrane conductance to ion Y is increased.

8. The relationship between contraction velocity and force for five different

skeletal muscles is shown. Which of the following muscles (A–E) is

most likely to correspond to muscle number 1 on the figure shown?

(Assume that all muscles shown are at their normal resting lengths.)


9. The diagram shows the relationship between muscle tension and

sarcomere length for skeletal muscle. Which point on the curve

represents tension development at a normal resting length?

A) A

B) B

C) C

D) D

E) E

10. The following events occur in a skeletal muscle during a normal

contraction: (1) increased calcium concentration in sarcoplasm, (2)

activation of ryanodine receptor, (3) calcium release from terminal

cisternae, and (4) activation of dihydropyridine voltage sensor. Which

of the following best describes the correct temporal order of events for a

normal contraction in a skeletal muscle fiber?

A) 1, 2, 3, 4

B) 1, 4, 2, 3

C) 1, 3, 2, 4

D) 3, 2, 1, 4

E) 3, 1, 2, 4

F) 4, 2, 1, 3

G) 2, 1, 4, 3

H) 2, 3, 4, 1

I) 4, 2, 3, 1

11. A 64-year-old man undergoes general anesthesia to remove a tumor

from his colon. Within a few minutes following administration of a

halogenated anesthetic, the patient develops muscle rigidity and a

rectal temperature of 108°F. His heart rate is 105 beats/min and

respiration rate is 29 breaths/min. Which of the following is most likely

to be decreased in this patient compared with normal resting

conditions?

A) Anaerobic metabolism

B) Calcium binding to calsequestrin

C) CO2 production by muscles

D) Muscle temperature

E) O2 usage by muscles

12. Which of the following best describes the selectivity filter of a

potassium ion channel in bacteria?

A) Glutamate

B) Carbonyl oxygens

C) Oxygen radicals

D) Gluten

E) Glycine

F) Carbon dioxide

13. During the course of a nerve action potential (shown), a 10-mV

electrical stimulus is delivered at the time indicated by the arrow. In

response to the electrical stimulus, a second action potential will:

A) be identical to the first

B) have a higher amplitude

C) have a lower amplitude

D) not occur

E) have a slower velocity

F) have a faster velocity

14. Which of the following best describes myasthenia gravis (MG) and

Lambert-Eaton myasthenic syndrome (LEMS)?

A) MG, postsynaptic disease; LEMS, presynaptic disease

B) MG, presynaptic disease; LEMS, presynaptic disease

C) MG, postsynaptic disease; LEMS, postsynaptic disease

D) MG, presynaptic disease; LEMS, postsynaptic disease

15. Electrical coupling between adjacent cells in visceral smooth muscle

can be aributed to which of the following?

A) Dense bodies

B) Gap junctions

C) Intermediate fibers

D) Mechanical junctions

E) Potassium channels

16. A 45-year-old man goes to the local gym to lift weights. He begins by

bench-pressing 130 lb as a warm-up procedure and then gradually

increases the weight. Which of the following sets of changes occur as he

adds more weight?

Activation of motor units Frequency of motor nerve action potentials

A) Decreased Decreased

B) Decreased Increased

C) Decreased No change

D)

Increased Decreased

E)

Increased Increased

F)

Increased No change

17. Which of the following substances have a higher extracellular

concentration compared with the intracellular concentration?

A) Calcium and chloride

B) Potassium and sodium

C) Calcium and potassium

D) Potassium and proteins

E) Chloride and proteins

18. Which of the following allows smooth muscle to maintain a sustained

contraction with minimal energy usage compared to a similar level of

sustained contraction of skeletal muscle?

A) Dense body

B) Gap junctions

C) Intermediate filaments

D) Latch state

E )Syncytial nature

Questions 19–21

The table shows the concentrations of four ions across the plasma membrane

of a hypothetical cell. Use this table to answer the next three questions.

Intracellular (mM) Extracellular (mM)

140 K+ 5 K

+

12 Na

+ 145 Na

+

5 Cl−

125 Cl−

0.0001 Ca

2+ 5 Ca

2+

19. Which of the following best describes the equilibrium potential for Cl −

(in millivolts)?

A) 0

B) 170

C) −170

D) 85

E) −85

20. Which of the following best describes the equilibrium potential for K+

(in millivolts)?

A) 0

B) 176

C) −176

D) 88

E) −88

21. The net driving force is greatest for which ion when the membrane

potential of this cell is −85 millivolts?

A) Ca2+

B) Cl −

C) K+

D) Na+

22. A single contraction of skeletal muscle is most likely to be terminated

by which of the following actions?

A) Closure of the postsynaptic nicotinic acetylcholine receptor

B) Removal of acetylcholine from the neuromuscular junction

C) Removal of Ca2+

 from the terminal of the motor neuron

D) Removal of sarcoplasmic Ca2+

E) Return of the dihydropyridine receptor to its resting conformation

23. The resting potential of a myelinated nerve fiber is primarily

dependent on the concentration gradient of which of the following

ions?

A) Ca2+

B) Cl −

C) HCO3

D) K+

E) Na+

24. A neurotransmier activates its receptor on an ion channel of a neuron,

which causes the water-filled channel to open. When the channel is

open, ions move through the channel down their respective

electrochemical gradients. A change in membrane potential follows.

Which of the following best describes the type of channel and

mechanism of ion transport?

Type of Channel Mechanism of Transport

A) Ligand gated Primary active transport

B) Ligand gated Diffusion

C) Ligand gated Secondary active transport

D) Voltage gated Primary active transport

E) Voltage gated Diffusion

F) Voltage gated Secondary active transport

25. Which of the following decreases in length during the contraction of a

skeletal muscle fiber?

A) A band of sarcomere

B) I band of sarcomere

C) Thick filaments

D) Thin filaments

E) Z disks of sarcomere

26. Equilibrium potentials for three unknown ions are shown in the above

figure. Note that ions S and R are positively charged and that ion Q is

negatively charged. Assume that the cell membrane is permeable to all

three ions and that the cell has a resting membrane potential of −90

millivolts. Which of the following best describes the net movement of

the various ions across the cell membrane by passive diffusion?

Q − R

+ S

+

A)

Inward Inward Inward

B)

Inward Inward Outward

C)

Inward Outward Inward

D)

Inward Outward Outward

E) Outward Inward Inward

F) Outward Inward Outward

G) Outward Inward Outward

27. Weightlifting can result in a dramatic increase in skeletal muscle mass.

This increase in muscle mass is primarily aributable to which of the

following?

A) Fusion of sarcomeres between adjacent myofibrils

B) Hypertrophy of individual muscle fibers

C) Increase in skeletal muscle blood supply

D) Increase in the number of motor neurons

E) Increase in the number of neuromuscular junctions

28. Five hypothetical nerve axons are shown in the above figure. Axons A

and B are myelinated, whereas axons C, D, and E are nonmyelinated.

Which axon is most likely to have the fastest conduction velocity for an

action potential?

A) A

B) B

C) C

D) D

E) E

Questions 29 and 30

The figure below shows the change in membrane potential during an action

potential in a giant squid axon. Refer to it when answering the next two

questions.

29. Which of the following is primarily responsible for the change in

membrane potential between points B and D?

A) Inhibition of the Na+

, K+ -ATPase

B) Movement of K+

 into the cell

C) Movement of K+

 out of the cell

D) Movement of Na+

 into the cell

E) Movement of Na+

 out of the cell

30. Which of the following is primarily responsible for the change in

membrane potential between points D and E?

A) Inhibition of the Na+

, K+-ATPase

B) Movement of K+

 into the cell

C) Movement of K+

 out of the cell

D) Movement of Na+

 into the cell

E) Movement of Na+

 out of the cell

31. The axon of a neuron is stimulated experimentally with a 25-millivolt

pulse, which initiates an action potential with a velocity of 50 m per

second. The axon is then stimulated with a 100-millivolt pulse. What is

the action potential velocity after the 100-millivolt stimulation pulse (in

meters per second)?

A) 25

B) 50

C) 100

D) 150

E) 200

Questions 32 and 33

The figure below illustrates the single isometric twitch characteristics of two

skeletal muscles, A and B, in response to a depolarizing stimulus. Refer to it

when answering the next two questions.

32. Which of the following best describes muscle B compared with muscle

A?

A) Adapted for rapid contraction

B) Composed of larger muscle fibers

C) Fewer mitochondria

D) Innervated by smaller nerve fibers

E) Less extensive blood supply

33. The delay between the termination of the transient depolarization of

the muscle membrane and the onset of muscle contraction observed in

both muscles A and B reflects the time necessary for which of the

following events to occur?

A) ADP to be released from the myosin head

B) ATP to be synthesized

C) Ca2+

 to accumulate in the sarcoplasm

D) G-actin to polymerize into F-actin

E) Myosin head to complete one cross-bridge cycle

Questions 34 and 35

A 32-year-old woman visits her physician because of double vision, eyelid

droop, difficulty chewing and swallowing, and general weakness in her limbs.

All these symptoms worsen with exercise and occur more frequently late in the

day. The physician suspects myasthenia gravis and orders a Tensilon test. The

test is positive. Use this information when answering the next two questions.

34. The increased muscle strength observed during the Tensilon test is due

to an increase in which of the following?

A) Amount of acetylcholine (ACh) released from the motor nerves

B) Levels of ACh at the muscle end-plates

C) Number of ACh receptors on the muscle end-plates

D) Synthesis of norepinephrine

35. Which of the following drugs would likely alleviate this patient’s

symptoms?

A) Atropine

B) Botulinum toxin antiserum

C) Curare

D) Halothane

E) Neostigmine

Questions 36–38

The figure below illustrates the isometric length–tension relationship in a

representative intact skeletal muscle. Match the descriptions in the next three

questions to one of the points on the figure.

36. So-called “active” or contraction-dependent tension

37. The muscle length at which active tension is maximal

38. The contribution of noncontractile muscle elements to total tension

39. Smooth muscle contraction is terminated by which of the following?

A) Dephosphorylation of myosin kinase

B) Dephosphorylation of myosin light chain

C) Efflux of Ca2+

 ions across the plasma membrane

D) Inhibition of myosin phosphatase

E) Uptake of Ca2+

 ions into the sarcoplasmic reticulum

Questions 40 and 41

A 73-year-old man sees a neurologist because of weakness in his legs that

improves over the course of the day or with exercise. Extracellular electrical

recordings from a single skeletal muscle fiber reveal normal miniature endplate potentials. Low-frequency electrical stimulation of the motor neuron,

however, elicits an abnormally small depolarization of the muscle fibers. The

amplitude of the depolarization is increased after exercise. Use this information

to answer the next three questions.

40. Based on these findings, which of the following is the most likely cause

of this patient’s leg weakness?

A) Acetylcholinesterase deficiency

B) Blockade of postsynaptic acetylcholine receptors

C) Impaired presynaptic voltage-sensitive Ca2+

 influx

D) Inhibition of Ca2+

 re-uptake into the sarcoplasmic reticulum

E) Reduced acetylcholine synthesis

41. A preliminary diagnosis is confirmed by the presence of which of the

following?

A) Antibodies against the acetylcholine receptor

B) Antibodies against the voltage-sensitive Ca2+

 channel

C) Mutation in the gene that codes for the ryanodine receptor

D) Relatively few vesicles in the presynaptic terminal

E) Residual acetylcholine in the neuromuscular junction

42. The molecular mechanism underlying these symptoms is most like

which of the following?

A) Acetylcholine

B) Botulinum toxin

C) Curare

D) Neostigmine

E) Tetrodotoxin

Questions 43–45

Match each of the descriptions in the next three questions to one of the

points of the nerve action potential shown in the figure.

43. Point at which the membrane potential (Vm) is closest to the Na+

equilibrium potential

44. Point at which the driving force for Na+

 is the greatest

45. Point at which the ratio of K+

 permeability to Na+

 permeability (PK

/PNa)

is the greatest

46. A physiology experiment is conducted in which a motoneuron that

normally innervates a predominantly fast Type II muscle is

anastomosed to a predominantly slow Type I muscle. Which of the

following is most likely to decrease in the Type I muscle after the

transinnervation surgery?

A) Fiber diameter

B) Glycolytic activity

C) Maximum contraction velocity

D) Mitochondrial content

E) Myosin ATPase activity

47. In the experiment illustrated in part A of the figure, equal volumes of

solutions X, Y, and Z are placed into the compartments of the two Ushaped vessels shown. The two compartments of each vessel are

separated by semipermeable membranes (i.e., membranes that are

impermeable to ions and large polar molecules). Part B illustrates the

fluid distribution across the membranes at equilibration. Assuming

complete dissociation, identify each of the solutions shown.

Solution X Solution Y Solution Z

A) 1 M CaCl2 1 M NaCl 1 M glucose

B) 1 M glucose 1 M NaCl 1 M CaCl2

C) 1 M NaCl 2 M glucose 3 M CaCl2

D) 2 M NaCl 1 M NaCl Pure water

E) Pure water 1 M CaCl2 2 M glucose

Questions 48 and 49

Use the figure shown below for the next two questions.

48. Trace A in the figure represents a typical action potential recorded

under control conditions from a normal neuron in response to a

depolarizing stimulus. Which of the following perturbations would

explain the conversion of the response shown in trace A to the action

potential shown in trace B?

A) Blockade of voltage-sensitive Na+

 channels

B) Blockade of voltage-sensitive K+

 channels

C) Blockade of Na-K “leak” channels

D) Replacement of the voltage-sensitive K+

 channels with “slow” Ca2+

channels

E) Replacement of the voltage-sensitive Na+

 channels with “slow”

Ca2+

 channels

49. Which of the following perturbations would account for the failure of

the same stimulus to elicit an action potential in trace C?

A) Blockade of voltage-sensitive Na+ channels

B) Blockade of voltage-sensitive K+ channels

C) Blockade of Na-K “leak” channels

D) Replacement of the voltage-sensitive K+

 channels with “slow” Ca2+

channels

E) Replacement of the voltage-sensitive Na+

 channels with “slow”

Ca2+

 channels

50. A 16-year-old soccer player sustained a fracture to the left tibia. After

her lower leg has been in a cast for 8 weeks, she is surprised to find that

the left gastrocnemius muscle is significantly smaller in circumference

than it was before the fracture. What is the most likely explanation?

A) Decrease in the number of individual muscle fibers in the left

gastrocnemius

B) Decrease in blood flow to the muscle caused by constriction from

the cast

C) Temporary reduction in actin and myosin protein synthesis

D) Increase in glycolytic activity in the affected muscle

E) Progressive denervation

Questions 51–55

Match each of the processes described in the next five questions with the

correct type of transport listed. Answers may be used more than once.

A) Diffusion

B) Exocytosis

C) Primary active transport

D) Co-transport

E) Counter-transport

51. Ouabain-sensitive transport of Na+

 ions from the cytosol to the

extracellular fluid

52. Glucose uptake into skeletal muscle

53. Na+-dependent transport of Ca2+

 from the cytosol to the extracellular

fluid

54. Transport of glucose from the intestinal lumen into an intestinal

epithelial cell

55. Movement of Na+

 ions into a neuron during the upstroke of an action

potential

56. Traces A, B, and C in the figure above summarize the changes in

membrane potential (Vm) and the underlying membrane permeabilities

(P) that occur in a neuron over the course of an action potential. Choose

the combination below that identifies each of the traces.

Trace A Trace B Trace C

A) PK Vm PNa

B) PK:PNa Vm PK

C) PNa Vm PK

D) Vm PK PNa

E) Vm PNa PK

57. A 45-year-old woman is admied as an emergency to University

Hospital after an automobile accident in which severe lacerations to the

left wrist severed a major muscle tendon. The severed ends of the

tendon were overlapped by 6 mm to facilitate suturing and

reaachment. Which of the following would be expected after 3 weeks

compared with the preinjured muscle? Assume that series growth of

sarcomeres cannot be completed within 3 weeks.

Passive Tension Maximal Active Tension

A) Decrease Decrease

B) Decrease Increase

C)

Increase Increase

D)

Increase Decrease

E) No change No change

58. The length–tension diagram above was obtained from a skeletal

muscle with equal numbers of red and white fibers. Supramaximal

tetanic stimuli were used to initiate an isometric contraction at each

muscle length studied. The resting length was 20 cm. What is the

maximum amount of active tension that the muscle can generate at a

preload of 100 grams?

A) 145 to 155 g

B) 25 to 35 g

C) 55 to 65 g

D) 95 to 105 g

E) Cannot be determined

59. The sensitivity of the smooth muscle contractile apparatus to calcium

is known to increase in the steady state under normal conditions. This

increase in calcium sensitivity can be aributed to a decrease in the

levels of which of the following substances?

A) Actin

B) Adenosine triphosphate (ATP)

C) Calcium–calmodulin complex

D) Calmodulin

E) Myosin light chain phosphatase (MLCP)

60. Which of the following best describes a physiological difference

between the contraction of smooth muscle compared with the

contraction of cardiac muscle and skeletal muscle?

A) Ca2+

 independent

B) Does not require an action potential

C) Requires more energy

D) Shorter in duration

61. The figure on the right column shows the force–velocity relationship

for contractions of skeletal muscle. The differences in the three curves

result from differences in which of the following?

A) Frequency of muscle contraction

B) Hypertrophy

C) Muscle mass

D) Myosin ATPase activity

E) Recruitment of motor units

Answers

1. D) The electrical current through a sodium channel caused by movement of

sodium ions is close to zero picoamperes when the channel is closed; the

current increases markedly when the channel is open. Only choice D shows a

low current (0.4 picoamperes) during the closed phase and a higher current

(3.1 picoamperes) during the open phase. It is not necessary to know the

open time of the channel to answer this question correctly; however, an open

time of 0.4 milliseconds is typical.

TMP14 pp. 54–55

2. B) A 150-millimolar solution of a solute has an osmolarity of 150

milliosmoles when the solute molecule does not dissociate. However, NaCl

dissociates into two molecules. In the human body with a typical

temperature of 37°C, about 93% of NaCl molecules are dissociated at any

given time (i.e., the dissociation constant is 0.93). Therefore, 150 millimoles

NaCl × 2 = 300 milliosmoles (without dissociation), and 300 milliosmoles x

0.93 = 279 milliosmoles (with a dissociation constant of 0.93).

TMP14 p. 58

3. E) The equilibrium potential of an ion can be calculated using the Nernst

equation as follows: Eion (in millivolts) = ± 61 × log (intracellular

concentration/extracellular concentration). The intracellular concentration of

potassium is relatively high compared to the extracellular concentration in

most cells of the body; this causes potassium to have a negative equilibrium

potential that averages about −90 millivolts in a typical neuron. A decrease in

extracellular potassium concentration (with no change in intracellular

concentration) would cause the potassium equilibrium potential to become

even more negative, according to the Nernst equation. The resting

membrane potential therefore would also become more negative because

this is dictated by the potassium equilibrium potential in normal cells of the

body.

TMP14 pp. 56–57

4. B) Sodium and chloride are nonpermenant ions that cannot move readily

through a lipid bilayer in either direction; this is typical of all charged ions

including potassium, calcium, bicarbonate, and hydrogen ions and others.

Because side Z in the figure has a greater initial concentration of NaCl

molecules compared with side Y, water will move down its concentration

gradient by osmosis from side Y to side Z, which will cause a decrease in the

volume of side Y and an increase in the volume of side Z. The total volume

of sides Y and Z together will not change, which excludes choices D and E.

TMP14 pp. 56, 58

5. A) Urea is a permeant molecule that can move through a lipid bilayer in

either direction. Hence, the concentration of urea will become equal in

compartments Y and Z within a fraction of a second. Water molecules can

more through the membrane more rapidly compared with urea, so the

volume in compartment Z will increase transiently. But again, the

concentration of urea will become equal on both sides of the membrane

within a fraction of a second, so the water volume will be the same in

compartments Y and Z at equilibrium.

TMP14 pp. 56, 58

6. D) Under basal conditions, the intracellular concentrations of sodium,

calcium, and chloride are less than the extracellular concentrations, whereas

potassium has a higher intracellular concentration compared with its

extracellular concentration. Transporter Y in the figure moves both

potassium and sodium ions against their concentration gradients, which is

primary active transport and is powered by ATP at the pump. Therefore,

when transporter Y is inhibited by ouabain, the intracellular concentration of

sodium increases (and the intracellular concentration of potassium

decreases). This increase in intracellular sodium concentration decreases the

sodium concentration gradient across the cell membrane. Now because the

energy required to move calcium ions out of the cell is powered by the

sodium concentration gradient (via secondary active transport), a decrease in

the transmembrane sodium gradient leads to an increase in the intracellular

calcium concentration. So, inhibition of transporter Y leads to increases in the

intracellular concentrations of both sodium and calcium ions. Cardiac

glycosides increase intracellular calcium concentration in cardiac muscle

cells by this mechanism.

TMP14 pp. 59–61

7. B) The resting membrane potential of a typical cell in the body is closest to

the equilibrium potential of the ion with the highest conductance (i.e.,

permeability). In most cells of the body, the conductance to potassium is

relatively high, causing the membrane potential to approach the potassium

equilibrium potential. In the diagram shown, the initial membrane potential

(Em) is represented by the level of line C. When the membrane conductance

to ion Y is increased, the membrane potential approaches the equilibrium

potential of ion Y; this eliminates answers C, D, and E. Answer A can also be

eliminated because the membrane potential cannot become greater than the

equilibrium potential of the ion.

TMP14 pp. 66–67

8. D) The maximum velocity of shortening of a muscle is dependent on the

predominant type of muscle fiber in the muscle as well as the overall length

of the muscle. In general, type II glycolytic muscles have a higher maximum

velocity of shortening compared with type I oxidative muscles. However, the

student must assume that all the muscles shown have similar proportions of

type I and type II fibers because this was not stated in the problem. Because

muscle 1 in the diagram has the second highest maximum velocity of

shortening, it corresponds with the second longest muscle (muscle D) shown

in the answer choices. Also, muscle D has the greatest diameter and thus

corresponds to Muscle 1, which is shown to exert the greatest force at zero

conduction velocity. Muscle diameter does not affect the maximum velocity

of shortening because this is extrapolated to a force of 0.

TMP14 pp. 85–86

9. C) A typical resting sarcomere length of 2 micrometers in skeletal muscle

provides optimal overlap of actin and myosin filaments, and thus, the

development of muscle tension is maximal at the resting length.

TMP14 p. 85

10. I) A normal contraction of a skeletal muscle fiber begins with depolarization

of the muscle fiber membrane which activates dihydropyridine voltage

sensors (event 4) of the transverse tubules. Activation of the dihydropyridine

voltage sensor leads to activation of the ryanodine receptor (event 2) with

subsequent release of calcium from the terminal cisternae (event 3); this

release of calcium increases the calcium concentration in the sarcoplasm

(event 1). Contraction of the muscle fiber follows.

TMP14 pp. 97–99

11. B) Malignant hyperthermia is a pharmacogenetic disorder of skeletal

muscle in which ryanodine receptors respond to certain halogenated

anesthetics (as well as the muscle relaxant succinylcholine) by opening their

associated calcium channels within the muscle fiber and thus causing an

increase in myoplasmic calcium. This increase in myoplasmic calcium

concentration causes continual contraction of the skeletal muscles

everywhere in the body. The results are increased body temperature,

increased anaerobic metabolism, increased CO2

 production, and increased

O2

 usage by all skeletal muscles. Calsequestrin is a protein molecule that

binds calcium within the sarcoplasmic reticulum of the muscle fiber. Because

calcium is continually leaking from the sarcoplasmic reticulum, the binding

of calcium to calsequestrin is decreased during an episode of malignant

hyperthermia.

TMP14 pp. 98, 100

12. B) Potassium channels in bacteria were found to have a tetrameric structure

consisting of four identical protein subunits surrounding a central pore. At

the top of the channel pore are pore loops that form a narrow selectivity filter.

Lining the selectivity filter are carbonyl oxygens. When hydrated potassium

ions enter the selectivity filter, they interact with the carbonyl oxygens and

shed most of their bound water molecules, permiing the dehydrated

potassium ions to pass through the channel. The carbonyl oxygens are too

far apart, however, to enable them to interact closely with the smaller

sodium ions, which are therefore effectively excluded by the selectivity filter

from passing through the pore.

TMP14 p. 53

13. D) A new action potential cannot occur in an excitable fiber when the

membrane is still depolarized from the preceding action potential. The

reason for this restriction is that shortly after the action potential is initiated,

the sodium channels (or calcium channels, or both) become inactivated, and

no amount of excitatory signal applied to these channels at this point will

open the inactivation gates. The only condition that will allow them to

reopen is for the membrane potential to return to or near the original resting

membrane potential level. Then, within another small fraction of a second,

the inactivation gates of the channels open and a new action potential can be

initiated.

TMP14 p. 76

14. A) Both myasthenia gravis (MG) and Lambert-Eaton myasthenic syndrome

(LEMS) can cause muscle weakness. In MG, antibodies aack the

acetylcholine receptors on the postsynaptic muscle fiber membrane. Damage

to the acetylcholine channels results in small endplate potentials that do not

reach a threshold value required for generation of an action potential in the

muscle fiber. In LEMS, antibodies aack the voltage-gated calcium channels

on the presynaptic membrane; without proper function of these channels,

insufficient amounts of acetylcholine are released into the neuromuscular

junction, again, resulting in small endplate potentials.

TMP14 p. 97

15. B) In visceral smooth muscle, the cell membranes are joined by many gap

junctions through which ions can flow freely from one muscle cell to the next

so that action potentials, or simple ion flow without action potentials, can

travel from one fiber to the next and cause the muscle fibers to contract

together at the same time. This type of smooth muscle is also known as

syncytial smooth muscle because of its syncytial interconnections among fibers.

It is also called visceral smooth muscle because it is found in the walls of most

viscera of the body, including the gastrointestinal tract, bile ducts, ureters,

uterus, and many blood vessels.

TMP14 pp. 105–106

16. E) Summation of muscle contractions occurs when a person aempts to lift

heavy weights. Summation occurs in two ways: (1) by increasing the number

of motor units contracting simultaneously, which is called multiple fiber

summation, and (2) by increasing the frequency of contraction of individual

muscle fibers, which is called frequency summation. So, when a person

aempts to lift a heavy weight, an increased number of motor units is

activated, and, the frequency of motor nerve action potentials to the motor

units of the muscle is also increased.

TMP14 pp. 88–89

17. A) The extracellular fluid contains a large amount of sodium, calcium, and

chloride but only a small amount of potassium. The opposite is true of the

intracellular fluid. However, the concentrations of phosphates and proteins

in the intracellular fluid are considerably greater than those in the

extracellular fluid. These differences are extremely important to the life of

the cell, as discussed inChapter 4.

TMP14 p. 51

18. D) When smooth muscle has developed full contraction, the amount of

continuing excitation can usually be reduced to far less than the initial level

even though the muscle maintains its full force of contraction. This

mechanism is called the “latch” mechanism.

TMP14 p. 103

19. E) The equilibrium potential for chloride (ECl −) can be calculated using the

Nernst equation as follows: ECl − (in millivolts) = +61 × log (Ci

/Co

), where Ci

is the intracellular concentration and Co

 is the extracellular concentration.

Hence, ECl − = +61 × log (5/125) = −85 millivolts.

TMP14 p. 63

20. E) The equilibrium potential for potassium (EK

+

) can be calculated using

the Nernst equation as follows: EK

+

 (in millivolts) = −61 × log (Ci

/Co

). In this

problem, EK

+

 = −61 × log (140/5) = −88 millivolts.

TMP14 p. 63

21. A) The net driving force on any ion is the difference in millivolts between

the membrane potential (Vm) and the equilibrium potential for that ion

(Eion). In this cell, EK

+

 = −88 millivolts, ECl − = −85 millivolts, ENa

+

 = +66

millivolts, and ECa

2+ = +145 millivolts. Therefore, Ca2+ is the ion with the

equilibrium potential farthest from Vm. This means that Ca2+ would have the

greatest tendency to cross the membrane and enter the cell through an open

channel in this hypothetical cell.

TMP14 p. 63

22. D) Skeletal muscle contraction is tightly regulated by the concentration of

Ca2+ in the sarcoplasm. As long as sarcoplasmic Ca2+ is sufficiently high,

none of the remaining events—removal of acetylcholine from the

neuromuscular junction, removal of Ca2+ from the presynaptic terminal,

closure of the acetylcholine receptor channel, and return of the

dihydropyridine receptor to its resting conformation—would have any effect

on the contractile state of the muscle.

TMP14 pp. 97–98

23. D) The resting potential of any cell is dependent on the concentration

gradients of the permeant ions and their relative permeabilities (Goldman

equation). In the myelinated nerve fiber, as in most cells, the resting

membrane is predominantly permeable to K+

. The negative membrane

potential observed in most cells (including nerve cells) is due primarily to

the relatively high intracellular concentration and high permeability of K+

.

TMP14 p. 64

24. B) A neurotransmier is considered to be a ligand, so when a

neurotransmier binds to its receptor on an ion channel, causing the channel

to open, the channel is said to be ligand gated; voltage-gated channels open

and close in response to changes in electrical potential across the cell

membrane. The mechanism of transport through all water-filled channels is

diffusion. Secondary active transport and primary active transport require

special transport proteins rather than water-filled channels in the membrane.

TMP14 p. 53

25. B) The physical lengths of the actin and myosin filaments do not change

during contraction. Therefore, the A band, which is composed of myosin

filaments, does not change either. The distance between Z disks decreases,

but the Z disks themselves do not change. Only the I band decreases in

length as the muscle contracts.

TMP14 p. 82

26. E) The equilibrium potential of an ion (also called the Nernst potential) is

the membrane potential at which there is no net movement of that ion across

the cell membrane. The various ions (Q, R, and S) move across the cell

membrane in the direction required to reach their individual equilibrium

potentials given the resting membrane potential of −90 millivolts. Negatively

charged Q ions must move out of the cell (outward) to achieve an

equilibrium potential of −75 millivolts (i.e., negatively charged ions must be

removed from the cell to cause the membrane potential to change from a

resting value of −90 millivolts to a value of −75 millivolts). Because the

positively charged R ion has an equilibrium potential of +75 millivolts, the R

ion must move into the cell to cause the membrane potential to change from

−90 millivolts to +75 millivolts. Ion S is a posi tively charged ion with an

equilibrium potential of −85 mV; this ion must move into the cell (inward) to

cause the membrane potential to change from −90 millivolts to −85 millivolts.

TMP14 pp. 64–65

27. B) Prolonged or repeated maximal contraction results in a concomitant

increase in the synthesis of contractile proteins and an increase in muscle

mass. This increase in mass, or hypertrophy, is observed at the level of

individual muscle fibers.

TMP14 pp. 90–91

28. B) The velocity of an action potential increases in proportion to the diameter

of the axon for both myelinated and nonmyelinated axons. Myelination

increases the velocity of an action potential by several orders of magnitude

more compared with the effect of an increase in axon diameter, which means

that a large myelinated axon has the highest velocity of conduction.

Therefore, even though unmyelinated axon E has the greatest diameter,

myelinated axon B can conduct an action potential at a much greater

velocity.

TMP14 pp. 74–75

29. D) At point B in this action potential, Vm has reached threshold potential

and has triggered the opening of voltage-gated Na+

 channels. The resulting

Na+

 influx is responsible for the rapid, self-perpetuating depolarization

phase of the action potential.

TMP14 pp. 67–68

30. C) The rapid depolarization phase is terminated at Point D by the

inactivation of the voltage-gated Na+

 channels and the opening of the

voltage-gated K+

 channels. The laer results in the efflux of K+

 from the

cytosol into the extracellular fluid and repolarization of the cell membrane.

TMP14 pp. 67–68

31. B) The velocity of an action potential is a function of the physical

characteristics of the axon (e.g., myelination, axon diameter). A given axon

will always conduct any action potential at the same velocity under normal

conditions. Therefore, stimulation of the axon with a 25-millivolt pulse or

100 millivolts will produce an action potential with the same velocity, which

is why action potentials are said to be “all or none.” However, the level of

stimulation must be sufficient to achieve a critical threshold level of potential

before an action potential can be initiated in an axon.

TMP14 p. 72

32. D) Muscle B is characteristic of a slow-twitch muscle (type I) composed of

predominantly slow-twitch muscle fibers. These fibers are smaller in size and

are innervated by smaller nerve fibers. They typically have a more extensive

blood supply, a greater number of mitochondria, and large amounts of

myoglobin, all of which support high levels of oxidative phosphorylation.

TMP14 p. 88

33. C) Muscle contraction is triggered by an increase in sarcoplasmic Ca2+

concentration. The delay between the termination of the depolarizing pulse

and the onset of muscle contraction, also called the “lag,” reflects the time

necessary for the depolarizing pulse to be translated into an increase in

sarcoplasmic Ca2+ concentration. This process involves a conformational

change in the voltage-sensing, or dihydropyridine receptor, located on the T

tubule membrane, along with the subsequent conformational change in the

ryanodine receptor on the sarcoplasmic reticulum and the release of Ca2+

from the sarcoplasmic reticulum.

TMP14 pp. 97–99

34. B) Myasthenia gravis is an autoimmune disease in which antibodies

damage postsynaptic nicotinic acetylcholine receptors. This damage prevents

the firing of an action potential in the postsynaptic membrane. Tensilon

(edrophonium) is a readily reversible acetylcholinesterase inhibitor that

increases acetylcholine levels in the neuromuscular junction, thereby

increasing the strength of muscle contraction.

TMP14 p. 97

35. E) Neostigmine is an acetylcholinesterase inhibitor. Administration of this

drug would increase the amount of ACh present in the synapse and its

ability to sufficiently depolarize the postsynaptic membrane and trigger an

action potential. Botulinum toxin antiserum is effective only against

botulinum toxicity. Curare blocks the nicotinic ACh receptor and causes

muscle weakness. Atropine is a muscarinic ACh receptor antagonist, and

halothane is an anesthetic gas. Neither atropine nor halothane has any effect

on the neuromuscular junction.

TMP14 pp. 96, 97

36. B) In this figure, “active” or contraction-dependent tension is the difference

between total tension (trace A) and the passive tension contributed by

noncontractile elements (trace C). The length-tension relationship in intact

muscle resembles the biphasic relationship observed in individual

sarcomeres and reflects the same physical interactions between actin and

myosin filaments.

TMP14 p. 85

37. E) “Active” tension is maximal at normal physiological muscle lengths. At

this point, there is optimal overlap between actin and myosin filaments to

support maximal cross-bridge formation and tension development.

TMP14 p. 85

38. C) Trace C represents the passive tension contributed by noncontractile

elements, including fascia, tendons, and ligaments. This passive tension

accounts for an increasingly large portion of the total tension recorded in

intact muscle as it is stretched beyond its normal length.

TMP14 p. 85

39. B) Smooth muscle contraction is regulated by both Ca2+ and myosin light

chain phosphorylation. When the cytosolic Ca2+ concentration decreases

after the initiation of contraction, myosin kinase becomes inactivated.

However, cross-bridge formation continues, even in the absence of Ca2+

,

until the myosin light chains are dephosphorylated through the action of

myosin light chain phosphatase.

TMP14 pp. 103–104

40. C) The normal miniature endplate potentials indicate sufficient synthesis

and packaging of ACh and the presence and normal function of ACh

receptor channels. The most likely explanation for this patient’s symptoms is

a presynaptic deficiency—in this case, an impairment of the voltage-sensitive

Ca2+ channels responsible for the increase in cytosolic Ca2+ that triggers the

release of ACh into the synapse. The increase in postsynaptic depolarization

observed after exercise is indicative of an accumulation of Ca2+ in the

presynaptic terminal after multiple action potentials have reached the nerve

terminal.

TMP14 pp. 93–94

41. B) Inhibition of the presynaptic voltage-sensitive Ca2+ channels is most

consistent with the presence of antibodies against this channel. Antibodies

against the ACh receptor, a mutation in the ryanodine receptor, and residual

ACh in the junction are all indicative of postsynaptic defects. Although it is a

presynaptic defect, a deficit of ACh vesicles is unlikely in this scenario, given

the normal miniature endplate potentials recorded in the postsynaptic

membrane.

TMP14 pp. 93–94

42. B) Botulinum toxin inhibits muscle contraction presynaptically by

decreasing the amount of ACh released into the neuromuscular junction. In

contrast, curare acts postsynaptically, blocking the nicotinic ACh receptors

and preventing the excitation of the muscle cell membrane. Tetrodotoxin

blocks voltage-sensitive Na+

 channels, affecting both the initiation and the

propagation of action potentials in the motor neuron. Both ACh and

neostigmine stimulate muscle contraction.

TMP14 pp. 95–96

43. D) During an action potential in a nerve cell, Vm approaches ENa during the

rapid depolarization phase when the permeability of the membrane to Na+

(PNa) increases relative to its permeability to K+

 (PK). In a “typical” cell, ENa

is close to 60 millivolts. Vm is closest to ENa at point D in this figure. At this

point, the ratio of PNa to PK is the greatest.

TMP14 pp. 70–71

44. F) The driving force for Na+

 is greatest at the point at which Vm is the

farthest from ENa. If ENa is very positive (≈ 60 millivolts), Vm is farthest from

ENa at point F, or when the cell is the most hyperpolarized.

TMP14 pp. 70–71

45. F) Generally, Vm is closest to the equilibrium potential of the most permeant

ion. In nerve cells, PK >> PNa at rest. As a result, Vm is relatively close to EK.

During the after-potential or the hyperpolarization phase of the action

potential, the ratio of PK to PNa is even greater than it is at rest because of the

residual opening of voltage-gated K+

 channels and the inactivation of the

voltage-gated Na+

 channels. PK:PNa is greatest at point F, at which point Vm

comes closest to EK.

TMP14 pp. 70–71

46. D) Muscle fibers have significant plasticity, which means that their

characteristics can change depending on the frequency at which they are

stimulated. When a nerve that innervates a predominantly fast type II

muscle is anastomosed to a predominantly slow type I muscle, the type I

muscle is converted to a type II muscle. Compared with type I muscle fibers,

type II fibers have a larger diameter, higher glycolytic activity, greater

maximum velocity of contraction, lower mitochondrial content, and higher

myosin ATPase activity. Therefore, only mitochondrial content decreases

when a type I fiber is converted to a type II fiber.

TMP14 p. 88

47. B) The redistribution of fluid volume shown in part B reflects the net

diffusion of water, or osmosis, because of differences in the osmolarity of the

solutions on either side of the semipermeable membrane. Osmosis occurs

from solutions of high water concentration to low water concentration or

from low osmolarity to high osmolarity. In part B, osmosis has occurred

from X to Y and from Y to Z. Therefore, the osmolarity of solution Z is higher

than that of solution Y, and the osmolarity of solution Y is higher than that of

solution X.

TMP14 pp. 57–58

48. E) These so-called slow Ca2+ channels have a slower inactivation rate,

thereby lengthening the time during which they are open. This phenomenon,

in turn, delays the repolarization phase of the action potential, creating a

“plateau” before the channels inactivate.

TMP14 pp. 72–73

49. A) In the absence of hyperpolarization, the inability of an otherwise

excitatory stimulus to initiate an action potential is most likely the result of

the blockade of the voltage-gated channels responsible for the generation of

the all-or-none depolarization. In nerve cells, these channels are the voltagegated Na+

 channels.

TMP14 pp. 68, 70

50. C) Skeletal muscle continuously remodels in response to its level of use.

When a muscle is inactive for an extended period, the rate of synthesis of the

contractile proteins in individual muscle fibers decreases, resulting in an

overall reduction in muscle mass. This reversible reduction in muscle mass is

called atrophy.

TMP14 pp. 90–91

51. C) Ouabain inhibits Na+

, K+

-ATPase. This ATP-dependent enzyme

transports three Na+

 ions out of the cell for every two K+

 ions it transports

into the cell. It is a classic example of primary active transport.

TMP14 p. 59

52. A) Glucose is transported into skeletal muscle cells via insulin-dependent

facilitated diffusion.

TMP14 p. 56

53. E) The activity of Na+

, K+

-ATPase maintains the relatively high K+

concentration inside the cell and the relatively high Na+

 concentration in the

extracellular fluid. This large concentration gradient for Na+

 across the

plasma membrane, together with the net negative charge on the inside of the

cell, continuously drives Na+

 ions from the extracellular fluid into the

cytosol. This energy is used to transport other molecules, such as Ca2+

 ,

against their concentration gradients. Because ATP is required to maintain

the Na+

 gradient that drives this counter-transport, this type of transport is

called secondary active transport.

TMP14 pp. 60–61

54. D) Much like Na+

- Ca2+ countertransport, the strong tendency for Na+

 to

move across the plasma membrane into the cytosol can be harnessed by

transport proteins and used to co-transport molecules against their

concentration gradients into the cytosol. An example of this type of

secondary co-transport is the transport of glucose into intestinal epithelial

cells.

TMP14 p. 61

55. A) During the rapid depolarization phase of a nerve action potential,

voltage-sensitive Na+

 channels open and allow the influx of Na+

 ions into the

cytosol. Transport through membrane channels is an example of diffusion.

TMP14 p. 68

56. E) Trace A exhibits the characteristic shape of an action potential, including

the rapid depolarization followed by a rapid repolarization that temporarily

overshoots the resting potential. Trace B best illustrates the change in PNa

that occurs during an action potential. The rapid increase in PNa closely

parallels the rapid depolarization phase of the action potential. Trace C best

illustrates the slow onset of the increase in PK that reflects the opening of the

voltage-gated K+

 channels.

TMP14 p. 70

57. D) Stretching the muscle to facilitate reaachment of the tendons leads to an

increase in passive tension or preload. This increase in passive tension

increases the muscle length beyond its ideal length, which in turn leads to a

decrease in the maximal active tension that can be generated by the muscle.

The reason maximal active tension decreases is that interdigitation of actin

and myosin filaments decreases when the muscle is stretched; the

interdigitation of a muscle is normally optimal at its resting length.

TMP14 p. 85

58. C) The figure shows the relationship between preload or passive tension

(curve Z), total tension (curve X), and active tension (curve Y). Active tension

cannot be measured directly: it is the difference between total tension and

passive tension. To answer this question, the student must first find where

100 g intersects the preload curve (passive tension curve) and then move

down to the active tension curve. One can see that a preload of 100 g is

associated with a total tension of a lile more than 150 g and an active

tension of a lile more than 50 g. Note that active tension equals total tension

minus passive tension, as previously discussed. Drawing these three curves

in a manner that is mathematically correct is not an easy task. The student

should thus recognize that active tension may not equal total tension minus

passive tension at all points on the figure shown here, as well as on United

States Medical Licensing Examination figures.

TMP14 p. 85

59. E) Smooth muscle is unique in its ability to generate various degrees of

tension at a constant concentration of intracellular calcium. This change in

calcium sensitivity of smooth muscle can be aributed to differences in the

activity of MLCP. Smooth muscle contracts when the myosin light chain is

phosphorylated by the actions of myosin light chain kinase (MLCK). MLCP

is a phosphatase that can dephosphorylate the myosin light chain, rendering

it inactive and therefore aenuating the muscle contraction. Choice A: Both

actin and myosin are important components of the smooth muscle

contractile apparatus, much like that of skeletal muscle and cardiac muscle,

but these components do not play a role in calcium sensitivity. Choice B:

ATP is required for smooth muscle contraction. Decreased ATP levels would

be expected to decrease the ability of smooth muscle to contract even in the

face of high calcium levels. Choice C: The calcium-–calmodulin complex

binds with MLCK, which leads to phosphorylation of the myosin light chain.

A decrease in the calcium- calmodulin complex should aenuate the

contraction of smooth muscle. Choice D: Again, the binding of calcium ions

to calmodulin is an initial step in the activation of the smooth muscle

contractile apparatus.

TMP14 p. 105

60. B) Smooth muscle can be stimulated to contract without the generation of

an action potential, whereas both cardiac muscle and skeletal muscle require

an action potential. Smooth muscle can contract in response to any stimulus

that increases the cytosolic Ca2+ concentration, which includes Ca2+ channel

openers, subthreshold depolarization, and a variety of tissue factors and

circulating hormones that stimulate the release of intracellular Ca2+ stores.

Smooth muscle contraction uses less energy and lasts longer compared with

that of skeletal muscle and cardiac muscle. Smooth muscle contraction is

heavily Ca2+ dependent.

TMP14 p. 108

61. D) The figure shows that the maximum velocity of shortening (Vmax) occurs

when there is no afterload on the muscle (force = 0). Increasing afterload

decreases the velocity of shortening until a point is reached at which

shortening does not occur (isometric contraction) and contraction velocity is

thus 0 (where curves intersect the x-axis). The maximum velocity of

shortening is dictated by the ATPase activity of the muscle, increasing to

high levels when the ATPase activity is elevated. Choice A: Increasing the

frequency of muscle contraction will increase the load that a muscle can lift

within the limits of the muscle, but it will not affect the velocity of

contraction. Choices B, C, and E: Muscle hypertrophy, increasing muscle

mass, and recruiting additional motor units will increase the maximum load

that a muscle can lift, but they will not affect the maximum velocity of

contraction.

TMP14 pp. 86, 88


Unit III: The Heart

Questions 1–4

A patient has a resting heart rate of 82 beats/min, normal blood

pressure, and normal body temperature. Use the pressure-volume

diagram of the left ventricle below to answer Questions 1–4.

1. What is the stroke volume in milliliters?

A) 150

B) 100

C) 85

D) 50

E) 70

2. What is the cardiac output of this patient?

A) 7000 ml/min

B) 50000 ml/min

C) 8200 ml/min

D) 8500 ml/min

E) 5000 ml/min

3. What is the extent of diastole in the ventricular pressure-volume

relationship?

A) At point B

B) From point D to point A

C) From point A to point C

D) From point D and point B

E) From point A and point B

4. What is correct about isovolumetric contraction?

A) Extends from B to C in the ventricular pressure-volume curve

B) Extends from D to A in the ventricular pressure-volume curve

C) Represents afterload in the ventricular pressure-volume curve

D) Represents a decrease in pressure with preserved volume

E) Depends on ventricular ejection

5. Which statement about action potential of cardiac muscle is most

accurate?

A) The calcium from T-tubules is less important as it is for

skeletal muscle

B) Phase 0 is predominantly dependent on slow potassium

channels

C) The end of the action potential (phase 2) causes an opening of

slow potassium channels

D) Action potential cause myofibrils to contract

E) Mucopolysaccharides inside the T-tubules provide chloride

ions to trigger phase 0

6. A 47-year-old man has an ejection fraction of 0.32 and an enddiastolic volume of 160 ml. What is (approximately) the value of

end-systolic volume?

A) 48 ml

B) 83 ml

C) 109 ml

D) 51 ml

E) 170 ml

7. In a resting adult, the typical ventricular ejection fraction has what

value?

A) 20%

B) 30%

C) 40%

D) 60%

E) 80%

8. In which phase of the ventricular muscle action potential is the

potassium permeability the highest?

A) 0

B) 1

C) 2

D) 3

E) 4

9. A 48-year-old man’s ECG shows that he has an R-R interval of 1.8

seconds at rest. Which statement best explains his condition?

A) He has fever

B) He may have an A-V block

C) He has decreased parasympathetic stimulation of the S-A

node

D) He is a trained athlete after exercise

E) He has augmented sympathetic stimulation of sinus node

10. Which of the following is most likely to cause the heart to go into

spastic contraction?

A) Increased body temperature

B) Increased sympathetic activity

C) Decreased extracellular fluid potassium ions

D) Excess extracellular fluid potassium ions

E) Excess extracellular fluid calcium ions

11. What happens at the end of ventricular isovolumic relaxation?

A) The A-V valves close

B) The aortic valve opens

C) The aortic valve closes

D) The mitral valve opens

E) The pulmonary valve closes

p y

12. Which event is associated with the first heart sound?

A) Closing of the aortic valve

B) Inrushing of blood into the ventricles during diastole

C) Beginning of diastole

D) Opening of the A-V valves

E) Closing of the A-V valves

13. Which condition will result in a dilated, flaccid heart?

A) Excess calcium ions in the blood

B) Excess potassium ions in the blood

C) Excess sodium ions in the blood

D) Increased sympathetic stimulation

E) Increased norepinephrine concentration in the blood

14. A 25-year-old well-conditioned athlete weighs 80 kg (176 lb).

During maximal sympathetic stimulation, what is the plateau level

of his cardiac output function curve?

A) 3 l/min

B) 5 l/min

C) 10 l/min

D) 13 l/min

E) 25 l/min

15. Which phase of the cardiac cycle follows immediately after the

beginning of the QRS wave?

A) Isovolumic relaxation

B) Ventricular ejection

C) Atrial systole

D) Diastasis

E) Isovolumic contraction

16. Which of the following structures will have the slowest rate of

conduction of the cardiac action potential?

A) Atrial muscle

B) Anterior internodal pathway

C) A-V bundle fibers

D) Purkinje fibers

E) Ventricular muscle

17. What is the normal total delay of the cardiac impulse in the A-V

node + bundle?

A) 0.22 second

B) 0.18 second

C) 0.16 second

D) 0.13 second

E) 0.09 second

18. Sympathetic stimulation of the heart does which of the following?

A) Releases acetylcholine at the sympathetic endings

B) Decreases sinus nodal discharge rate

C) Decreases excitability of the heart

D) Releases norepinephrine at the sympathetic endings

E) Decreases cardiac contractility

19. If the S-A node discharges at 0.00 seconds, when will the action

potential normally arrive at the epicardial surface at the base of the

left ventricle?

A) 0.22 second

B) 0.18 second

C) 0.16 second

D) 0.12 second

E) 0.09 second

20. Which condition at the A-V node will cause a decrease in heart

rate?

A) Increased sodium permeability

B) Decreased acetylcholine levels

C) Increased norepinephrine levels

D) Increased potassium permeability

E) Increased calcium permeability

21. Which statement best explains how sympathetic stimulation affects

the heart?

A) The permeability of the S-A node to sodium decreases

B) The permeability of the A-V node to sodium decreases

C) The permeability of the S-A node to potassium increases

D) There is an increased rate of upward drift of the resting

membrane potential of the S-A node

E) The permeability of the cardiac muscle to calcium decreases

22. What is the membrane potential (threshold level) at which the S-A

node discharges?

A) −40 millivolt

B) −55 millivolt

C) −65 millivolt

D) −85 millivolt

E) −105 millivolt

23. Which condition at the S-A node will cause the heart rate to

decrease?

A) Increased norepinephrine level

B) Increased sodium permeability

C) Increased calcium permeability

D) Increased potassium permeability

E) Decreased acetylcholine level

24. In which phase of the ventricular muscle action potential is the

sodium permeability the highest?

A) 0

B) 1

C) 2

D) 3

E) 4

25. If the S-A node discharges at 0.00 seconds, when will the action

potential normally arrive at the A-V bundle (bundle of His)?

A) 0.22 second

B) 0.18 second

C) 0.16 second

D) 0.12 second

E) 0.09 second

26. If the Purkinje fibers, situated distal to the A-V junction, become

the pacemaker of the heart, what is the expected heart rate?

A) 30/min

B) 50/min

C) 60/min

D) 70/min

E) 80/min

27. What is correct about the sinus node?

A) Delays the cardiac conduction if sympathetic activity

increases

B) Acts as a pacemaker because the membrane constantly leaks

Na+

 from extracellular fluid

C) The constant leak of K+

 makes resting potential in the sinus

node gradually rise

D) Feedback from the Purkinje fibers defines the sinus node

discharge

E) The resting membrane potential of the sinus node is +55 to

+60mV

28. What is correct about the conduction system?

A) The longest delay of the cardiac impulse is in the A-V bundles

B) The lack of GAP junctions is responsible for the fast

conduction of Purkinje fibers

C) The A-V node inhibit the sinus node during exercise

(overdrive suppression)

D) If sinus node fails, lower portions of the conduction system

can act as pacemaker

29. A patient had an ECG at the local emergency department. The

aending physician stated that the patient had an A-V nodal

rhythm. What is the likely heart rate?

A) 30/min

B) 50/min

C) 65/min

D) 75/min

E) 85/min

30. Which of the following is correct about ECG?

A) The mean vector of depolarization moves from negative to

positive, from front to back, from left to right

B) The P wave represents atrial depolarization and

repolarization

C) The Q-T interval approximates the time of ventricular

contraction

D) The P-R interval includes ventricular repolarization

E) The T wave always opposes QRS polarity

31. When recording lead aVL on an ECG, which is the positive

electrode?

A) Left arm

B) Left leg

C) Right leg

D) Left arm + left leg

E) Right arm + left leg

32. When recording lead II on an ECG, the right arm is the negative

electrode and the positive electrode is the

A) Left arm

B) Left leg

C) Right leg

D) Left arm + left leg

E) Right arm + left leg

g g

33. Sympathetic stimulation of the heart normally causes which

condition?

A) Acetylcholine release at the sympathetic endings

B) Decreased heart rate

C) Decreased rate of conduction of the cardiac impulse

D) Decreased force of contraction of the atria

E) Increased force of contraction of the ventricles

Questions 34 and 35

A 70-year-old woman had an ECG at her annual checkup. Use her lead

II recording below to answer Questions 34 and 35.

34. What is her heart rate in beats/min?

A) 70

B) 78

C) 84

D) 94

E) 104

35. According to Einthoven’s law, if the QRS voltage in lead III is 0.4

millivolt, what is the QRS voltage in lead I?

A) 0.05 millivolt

B) 0.50 millivolt

C) 1.05 millivolts

D) 1.25 millivolts

E) 2.05 millivolts

36. What is the normal QT interval?

A) 0.03 second

B) 0.13 second

C) 0.16 second

D) 0.20 second

E) 0.35 second

37. When recording lead II on an ECG, the negative electrode is the

A) Right arm

B) Left leg

C) Right leg

D) Left arm + left leg

E) Right arm + left leg

38. When recording aVF on an ECG, the negative electrode/s is/are in

which area?

A) Left arm, left leg

B) Right arm, left arm

C) Left leg

D) Left leg, right leg

E) Chest

39. A 65-year-old man had an ECG at a local emergency department

after a biking accident. His weight was 80 kg (176 lb), and his aortic

blood pressure was 160/90 mm Hg. The QRS voltage was 0.5

millivolt in lead I and 1.5 millivolts in lead III. What is the QRS

voltage in lead II?

A) 0.5 millivolt

B) 1.0 millivolt

C) 1.5 millivolts

D) 2.0 millivolts

E) 2.5 millivolts

40. What is correct about ECG leads?

A) The anterior or posterior origin of the current of injury is

defined by precordial (chest) leads

B) The anterior or posterior origin of the current of injury is

defined by bipolar leads

C) Displaced J point in aVF lead suggest lateral ischemia

D) ST elevation in aVR suggests ischemia in the apex of the left

ventricle

E) The T wave is always positive in all leads.

y p

Questions 41–43

A 60-year-old woman had an ECG recorded at a local emergency

department after an automobile accident. Her weight was 70 kg (154 lb),

and her aortic blood pressure was 140/80 mm Hg. Use this information

and the figure below to answer Questions 41–43.

41. What is the mean electrical axis calculated from standard leads I,

II, and III shown in the woman’s ECG?

A) −90 degrees

B) −50 degrees

C) −12 degrees

D) +100 degrees

E) +170 degrees

42. What is the heart rate using lead I for the calculation?

A) 70

B) 88

C) 100

D) 112

E) 148

43. What is her likely diagnosis?

A) Tricuspid valve stenosis

B) Left bundle branch block

C) Pulmonary valve stenosis

D) Pulmonary valve insufficiency

E) Aortic insufficiency

44. Which event is most often associated with deviation of electrical

axis to the left?

A) Pulmonary stenosis

B) Right bundle branch block

C) Aortic stenosis

D) Tetralogy of Fallot

E) Third degree A-V block

45. A ventricular depolarization wave, when traveling 60 degrees in

the frontal plane, will cause a large positive deflection in which of

the following leads?

A) aVR

B) aVL

C) Lead I

D) Lead II

E) aVF

A 50-year-old woman was admied to a local emergency department

after a motorcycle accident. The following ECG was obtained.

46. What is her heart rate? Use lead I for the calculation.

A) 56

B) 66

C) 76

D) 103

E) 152

47. Which condition or individual may show increased voltage of

ECG leads?

A) A 76-year old patient with old infarcts

B) A 37-year old patient with old infarcts

C) A trained athlete

D) Pericardial effusion (increased conductance)

E) Pleural effusion

48. Mr. Smith had an ECG at a local hospital, but his records were

lost. The ECG technician remembered that the QRS deflection was

large and positive in lead II and 0 in aVL. What is his mean

electrical axis in the frontal plane?

A) 90 degrees

B) 60 degrees

C) 0 degree

D) −60 degrees

E) −90 degrees

49. A 70-year-old woman came to a hospital emergency department

because she was experiencing chest pain. Based on the ECG shown

above, what is the likely diagnosis?

A) Acute anterior infarction in the left ventricle of the heart

B) Acute anterior infarction in the right ventricle of the heart

C) Acute posterior infarction in the left ventricle of the heart

D) Acute posterior infarction in the right ventricle of the heart

E) Right ventricular hypertrophy

50. A 55-year-old man underwent an ECG at an annual physical

examination and his net deflection (R wave minus Q or S wave) in

standard limb lead I was −1.2 millivolts. Standard limb lead II has a

net deflection of +1.2 millivolts. What is the mean electrical axis of

his QRS?

A) −30 degrees

B) +30 degrees

C) +60 degrees

D) +120 degrees

E) −120 degrees

51. During the T-P interval in an ECG of a patient with a damaged

cardiac muscle, which of the following is true?

A) The entire ventricle is depolarized

B) The entire ventricle is depolarized except for the damaged

cardiac muscle

C) About half the ventricle is depolarized

D) The entire ventricle is repolarized

E) The entire ventricle is repolarized except for the damaged

cardiac muscle

52. A 50-year-old man is a new employee at ABC Software. The above

ECG was recorded during a routine physical examination. What is

his likely diagnosis?

A) Chronic systemic hypertension

B) Chronic pulmonary hypertension

C) Second-degree heart block

D) Paroxysmal tachycardia

E) Tricuspid valve stenosis

53. A 30-year-old man had an ECG at his physician’s office, but his

records were lost. The ECG technician remembered that the QRS

deflection was large and positive in lead aVF and 0 in lead I. What

is the mean electrical axis in the frontal plane?

A) 90 degrees

B) 60 degrees

C) 0 degree

D) −60 degrees

E) −90 degrees

54. A 60-year-old woman tires easily. Her ECG shows a QRS complex

that is positive in the aVF lead and negative in standard limb lead

I. What is a likely cause of this condition?

A) Chronic systemic hypertension

B) Pulmonary hypertension

C) Aortic valve stenosis

D) Aortic valve regurgitation

55. A 65-year-old patient with a heart murmur has a mean QRS axis of

120 degrees, and the QRS complex lasts 0.18 second. What is the

likely diagnosis?

A) Aortic valve stenosis

B) Aortic valve regurgitation

C) Mitral valve regurgitation

D) Right bundle branch block

E) Left bundle branch block

56. A 60-year-old woman came to the hospital emergency department

and reported chest pain. Based on the ECG tracing shown above,

what is the most likely diagnosis?

A) Acute anterior infarction in the base of the heart

B) Acute anterior infarction in the apex of the heart

C) Acute posterior infarction in the base of the heart

D) Acute posterior infarction in the apex of the heart

E) Right ventricular hypertrophy

57. A 50-year-old man has been having fainting “spells” for about 2

weeks. During the episodes, his ECG shows a ventricular rate of 25

beats/min and 100 P waves/min. After about 30 seconds of fainting,

a normal sinus rhythm recurs. What is his likely diagnosis?

A) Atrial fluer

B) First-degree A-V block

C) Second-degree A-V block

D) Third-degree A-V block

E) Stokes-Adams syndrome

58. An 80-year-old man had an ECG taken at his local doctor’s office,

and the diagnosis was atrial fibrillation. Which condition is likely

in someone with atrial fibrillation?

A) Ventricular fibrillation, which normally accompanies atrial

fibrillation

B) Strong P waves on the ECG

C) An irregular and fast rate of ventricular contraction

D) A normal atrial “a” wave

E) A smaller atrial volume than normal

59. Circus movements in the ventricle can lead to ventricular

fibrillation. Which condition in the ventricular muscle will increase

the tendency for circus movements?

A) Decreased refractory period

B) Low extracellular potassium concentration

C) Increased refractory period

D) Shorter conduction pathway (decreased ventricular volume)

E) Increase in parasympathetic impulses to the heart

60. A 50-year-old man has a blood pressure of 140/85 mm Hg and

weighs 90.7 kg (200 lb). He reports that he is not feeling well, his

ECG has no P waves, he has a heart rate of 46 beats/min, and the

QRS complexes occur regularly. What is his likely condition?

A) First-degree heart block

B) Second-degree heart block

C) Third-degree heart block

D) Sinoatrial heart block

E) Sinus bradycardia

61. The following ECG tracing was obtained for a 60-year-old man

who weighs 99.8 kg (220 lb). Standard lead II is shown above. What

is his diagnosis?

A) A-V nodal rhythm

B) First-degree A-V heart block

C) Second-degree A-V heart block

D) Third-degree A-V heart block

E) Atrial fluer

62. A 35-year-old woman had unusual sensations in her chest after

she smoked a cigaree. Her ECG tracing is shown above. What is

the likely diagnosis?

A) Premature contraction originating in the atrium

B) Premature contraction originating high in the A-V node

C) Premature contraction originating low in the A-V node

D) Premature contraction originating in the apex of the ventricle

E) Premature contraction originating in the base of the ventricle

Questions 63 and 64

A 55-year-old man had the below ECG tracing recorded at his doctor’s

office at a routine physical examination. Use this tracing to answer

Questions 63 and 64.

63. What is his diagnosis?

A) Normal ECG

B) Atrial fluer

C) A high A-V junctional pacemaker

D) A middle A-V junctional pacemaker

E) A low A-V junctional pacemaker

64. What is his ventricular heart rate in beats/min?

A) 37.5

B) 60

C) 75

D) 100

E) 150

65. Which of the following is correct about ventricular fibrillation

(VF)?

A) Circus movements (a potential mechanism of VF) from the

atria are conducted to the ventricles and back

B) A potential trigger for a circus movements (a potential

mechanism of VF) is a longer refractory period

C) A potential trigger for a circus movements (a potential

mechanism of VF) is a decreased in velocity of conduction

D) VF usually reverses spontaneously

E) If heart rate is not over 120/min, VF does not need to be

treated

66. Which of the following will usually result in an inverted P wave

that occurs after the QRS complex?

A) Premature contraction originating in the atrium

B) Premature contraction originating high in the A-V junction

C) Premature contraction originating in the middle of the A-V

junction

D) Premature contraction originating low in the A-V junction

E) Atrial fibrillation

67. A 65-year-old woman who had a myocardial infarction 10 days

ago returned to her family physician’s office and reported that her

pulse rate felt rapid. Based on the above ECG tracing, what is the

likely diagnosis?

A) Stokes-Adams syndrome

B) Atrial fibrillation

C) A-V nodal tachycardia

D) Atrial paroxysmal tachycardia

E) Ventricular paroxysmal tachycardia

68. A 65-year-old man had the above ECG tracing recorded at his

annual physical examination. What is the likely diagnosis?

A) Atrial paroxysmal tachycardia

B) First-degree A-V block

C) Second-degree A-V block

D) Third-degree A-V block

E) Atrial fluer

69. A 60-year-old woman has been diagnosed with atrial fibrillation.

Which statement best describes this condition?

A) The ventricular rate of contraction is 140 beats/min

B) The P waves of the ECG are pronounced

C) Ventricular contractions occur at regular intervals

D) The QRS waves are more pronounced than normal

E) The atria are smaller than normal

70. What occurs after electrical shock of the heart with a 60-cycle

alternating current?

A) A normal arterial pressure

B) A decreased ventricular refractory period

C) Increased electrical conduction velocity

D) A shortened conduction pathway around the heart

E) Normal cardiac output

71. A 55-year-old man has been diagnosed with Stokes-Adams

syndrome. Two minutes after the syndrome starts to cause active

blockade of the cardiac impulse, which of the following is the

pacemaker of the heart?

A) Sinus node

B) A-V node

C) Purkinje fibers

D) Inter-atrial septum

E) Left atrium

72. The T wave represents ventricular repolarization. Which of the

following is correct about the T wave and repolarization?

A) Septum and endocardium are last to depolarize and first to

repolarize and results in negative T wave in V6

B) Repolarized areas will have a - charge first; therefore, a + net

vector occurs, and a flat T wave (points towards the apex)

develops

C) In healthy individuals, last area to repolarize is near the apex,

resulting in negative T waves in all chest leads

D) Ischemia can alter repolarization and induce T abnormalities

(e.g., flat T waves, taller, inverted)

E) Potassium levels are normally between 3.5 and 5.0 mEq/l.

Values higher than this can result in flat T waves and shorter

PR interval.

73. A man had a myocardial infarction at age 55 years. He is now 63

years old. Use the standard limb lead I tracing on his ECG shown

above to answer this question. What is his current diagnosis?

A) Sinus tachycardia

B) First-degree heart block

C) Second-degree heart block

D) ST segment depression

E) Third-degree heart block

74. Which statement best describes a patient with premature atrial

contraction?

A) The pulse taken from the radial artery immediately after the

premature contraction will be weak

B) Stroke volume immediately after the premature contraction

will be increased

C) The P wave is never seen

D) The probability of these premature contractions occurring is

decreased in people with a large caffeine intake

E) It causes the QRS interval to be lengthened

75. If the origin of the stimulus that causes atrial paroxysmal

tachycardia is near the A-V node, which statement about the P

wave in standard limb lead I is most accurate?

A) The P wave will originate in the sinus node

B) The P wave will be upright

C) The P wave will be inverted

D) The P wave will be missing

76. A 45-year-old man had the above ECG recorded at his annual

physical. What is the likely diagnosis?

A) Atrial paroxysmal tachycardia

B) First-degree A-V block

C) Second-degree A-V block

D) Ventricular paroxysmal tachycardia

E) Atrial fluer

77. Which of the following is a feature of premature ventricular

contractions (PVCs)?

A) The QRS is of shorter duration

B) P is always present

C) T shows opposing polarity to QRS

D) They only appear in healthy individuals

E) The PVC impulse always travels backward into atria and

forward to the ventricles

78. Which of the following is a feature of a sinoatrial block?

A) Cessation of normal P wave

B) Prolonged but fixed P-R interval

C) An increasing P-R interval and then a dropped beat

D) A fixed long P-R interval and then a dropped beat

E) A full compensatory pause

79. A 67-year old man with a history of hypertension has an ECG that

shows no P waves, irregular heart rate (86–112 beats/min), ab

electrical axis of +20 and QRS of 0.12 second. What is the most

likely interpretation of the ECG and diagnosis?

A) First degree A-V block and right bundle branch block

B) Atrial paroxysmal tachycardia left ventricular ischemia

C) Atrial fluer and left ventricular hypertrophy

D) Atrial fibrillation and left bundle branch block

E) Atrial fibrillation and left ventricular hypertrophy

Answers

1. B) Stroke volume (SV) of the heart is obtained by subtracting endsystolic volume (ESV, point D) from end-diastolic volume (EDV, point

B) for a given ventricle. SV=EDV-ESV

TMP14 pp. 119–120

2. C) Cardiac output can be calculated by multiplying SV times heart rate.

If stroke volume is 100 ml and heart rate is 82 beats/min, then 100 × 82 =

8200 ml/min.

TMP14 p. 125

3. D) Diastole includes the isovolumetric relaxation period (D to A) and

ventricular filling (A-AV valve opening- to B -AV valve closure-). Thus,

the diastole extends from point D to point B.

TMP14 p. 122

4. A) The isovolumetric contraction phase starts immediately after AV

valve closure and finishes when ejection phase starts. In this phase, there

is a build-up in intra-ventricular pressure without changes in volume

(both AV and aortic valves are closed). Thus, this period extends from

point B to point C.

TMP14 p. 122

5. D) The direct consequence of the action potential on cardiac muscle is

the contraction of the myofibrils, which is known as the

excitation/contraction coupling.

TMP14 p. 127

6. D) Ejection fraction (EF) is calculated as follows: stroke volume

(SV)/end-diastolic volume (EDV) multiplied by 100 and expressed in

percent. If the EF of this patients is 0.32, that means 32%. If the enddiastolic volume is 160, then 0.32 of that value will represent the stroke

volume (SV), and subtracting the SV from EDV will result in the endsystolic volume, which is exactly 108.8 ml (approximately 109 ml)

TMP14 p. 119

7. D) The typical ejection fraction is 60%, and lower values are indicative of

a weakened heart.

TMP14 p. 119

8. D) During phase 3 of the ventricular muscle action potential, the

potassium permeability of ventricular muscle greatly increases, which

causes a more negative membrane potential.

TMP14 p. 115

9. B) A R-R interval of 1.8 indicates that the heart beats every 1.8 seconds.

Using the formula of heart rate calculation (heart rate = 60 seconds/0.83,

which is the normal/average value of one beat every 0.83 seconds); the

heart rate of this person is 33 beats/min (60/1.8). A heart rate of 33

beats/min indicates bradycardia and is suggestive abnormal/blocked

conduction of the impulse.

TMP14 pp. 136, 157–159

10. E) The heart goes into spastic contraction after a large increase in the

calcium ion concentration surrounding the cardiac myofibrils, which

occurs if the extracellular fluid calcium ion concentration increases too

much. An excess potassium concentration in the extracellular fluids

causes the heart to become dilated because of the decrease in resting

membrane potential of the cardiac muscle fibers.

TMP14 p. 125

11. D) At the end of isovolumic relaxation, the mitral and tricuspid valves

open, which is followed by the period of diastolic filling.

p y p g

TMP14 pp.118, 121–122

12. E) The first heart sound by definition occurs just after the ventricular

pressure exceeds the atrial pressure, which causes the A-V valves to

mechanically close. The second heart sound occurs when the aortic and

pulmonary valves close.

TMP14 pp. 121–122

13. B) Having excess potassium ions in the blood and extracellular fluid

causes the heart to become dilated and flaccid and slows the heart. This

effect is important because of a decrease in the resting membrane

potential in the cardiac muscle fibers. As the membrane potential

decreases, the intensity of the action potential decreases, which makes

the contraction of the heart progressively weaker. Excess calcium ions in

the blood and sympathetic stimulation and increased norepinephrine

concentration of the blood all cause the heart to contract vigorously.

TMP14 p. 125

14. E) The normal plateau level of the cardiac output function curve is 13

L/min. This level decreases in any kind of cardiac failure and increases

markedly during sympathetic stimulation.

TMP14 p. 125

15. E) Immediately after the QRS wave, the ventricles begin to contract,

and the first phase that occurs is isovolumic contraction. Isovolumic

contraction occurs before the ejection phase and increases the

ventricular pressure enough to mechanically open the aortic and

pulmonary valves.

TMP14 p. 118

16. C) The atrial and ventricular muscles have a relatively rapid rate of

conduction of the cardiac action potential, and the anterior internodal

pathway also has fairly rapid conduction of the impulse. However, the

A-V bundle myofibrils have a slow rate of conduction because their

sizes are considerably smaller than the sizes of the normal atrial and

ventricular muscle. In addition, their slow conduction is partly caused

by diminished numbers of gap junctions between successive muscle

cells in the conducting pathway, causing a great resistance to

conduction of the excitatory ions from one cell to the next.

TMP14 p. 129

17. D) The impulse from the S-A node travels rapidly through the

internodal pathways and arrives at the A-V node at 0.03 second, at the

A-V bundle at 0.12 second, and at the ventricular septum at 0.16 second.

The total delay is thus 0.13 second.

TMP14 p. 129

18. D) Increased sympathetic stimulation of the heart increases heart rate,

atrial contractility, and ventricular contractility and increases

norepinephrine release at the ventricular sympathetic nerve endings. It

does not release acetylcholine. It does cause an increased sodium

permeability of the A-V node, which increases the rate of upward drift

of the membrane potential to the threshold level for self-excitation, thus

increasing the heart rate.

TMP14 pp. 123–125, 132

19. A) After the S-A node discharges, the action potential travels through

the atria, through the A-V bundle system, and finally to the ventricular

septum and throughout the ventricle. The last place that the impulse

arrives is at the epicardial surface at the base of the left ventricle, which

requires a transit time of 0.22 second.

TMP14 p. 130

20. D) The increase in potassium permeability causes a hyperpolarization

of the A-V node, which will decrease the heart rate. Increases in sodium

permeability actually partially depolarizes the A-V node, and an

increase in norepinephrine levels increases the heart rate.

TMP14 p. 132

21. D) During sympathetic stimulation, the permeabilities of the S-A node

and the A-V node increase. In addition, the permeability of cardiac

muscle to calcium increases, resulting in an increased contractile

strength. Furthermore, an upward drift of the resting membrane

potential of the S-A node occurs. Increased permeability of the S-A node

to potassium does not occur during sympathetic stimulation.

TMP14 p. 132

22. A) The normal resting membrane potential of the S-A node is −55

millivolts. As the sodium leaks into the membrane, an upward drift of

the membrane potential occurs until it reaches −40 millivolts. This is the

threshold level that initiates the action potential at the S-A node.

TMP14 pp. 127–129

23. D) Increases in sodium and calcium permeability at the S-A node result

in an increase in heart rate. An increased potassium permeability causes

a hyperpolarization of the S-A node, which causes the heart rate to

decrease.

TMP14 p. 132

24. A) Sodium permeability is highest during phase 0. Calcium

permeability is highest during phase 2, and potassium is most

permeable in phase 3.

TMP14 p. 114–116

25. D) The action potential arrives at the A-V bundle at 0.12 second. It

arrives at the A-V node at 0.03 second and is delayed 0.09 second in the

A-V node, which results in an arrival time at the bundle of His of 0.12

second.

TMP14 p. 130

26. A) If the Purkinje fibers are the pacemaker of the heart, the heart rate

ranges between 15 and 40 beats/min. In contrast, the rate of firing of the

A-V nodal fibers are 40 to 60 times a minute, and the sinus node fires at

70 to 80 times/min. If the sinus node is blocked for some reason, the A-V

node will take over as the pacemaker, and if the A-V node is blocked,

the Purkinje fibers will take over as the pacemaker of the heart.

TMP14 p. 131

27. B) The inherent and constant leakiness to Na+

 (and movement of Ca2+)

are responsible for the automatic discharge (self-excitation) of the sinus

node because it makes resting potential to gradually rise to the point

(about −40 mV) of triggering the action potential.

TMP14 p. 128

28. D) If sinus node discharge does not get through, the next fastest area of

discharge becomes the pacemaker of the heart beat but at a lower

discharge rate because of a gradually more negative resting potential.

The new pacemaker may be the AV node or penetrating part of AV

bundle, and if these regions fail, Purkinje fibers will take the lead as the

pacemaker of the heart.

TMP14 p. 131

29. B) The normal rhythm of the A-V node is 40 to 60 beats/min. Purkinje

fibers have a rhythm of 15 to 40 beats/min.

TMP14 p. 131

30. C) The QRS complex represents ventricular depolarization, and the T

wave represents ventricular repolarization. The Q-T interval is the time

that the ventricle takes to depolarize-repolarize and represents the ven

tricular contraction, as observed in the Wiggers’ diagram.

TMP14 pp. 135–137

31. A) By convention, the left arm is the positive electrode for lead aVL of

an ECG.

TMP14 p. 139

32. B) By convention, the left leg is the positive electrode for lead II of an

ECG.

TMP14 p. 139

33. E) Sympathetic stimulation of the heart normally causes an increased

heart rate, increased rate of conduction of the cardiac impulse, and

increased force of contraction in the atria and ventricles. However, it

does not cause acetylcholine release at the sympathetic endings because

they contain norepinephrine. Parasympathetic stimulation causes

acetylcholine release. The sympathetic nervous system firing increases

in the permeability of the cardiac muscle fibers, the S-A node, and the AV node to sodium and calcium.

TMP14 p. 132

34. A) The heart rate can be calculated by 60 divided by the R-R interval,

which is 0.86 second. This results in a heart rate of 70 beats/min.

TMP14 p. 137

35. B) Einthoven’s law states that the voltage in lead I plus the voltage in

lead III is equal to the voltage in lead II. In this case, the voltage in lead

II is 0.9 millivolt, and the voltage in lead III is 0.4 millivolt. The lead I

voltage is thus 0.5 (0.9 − 0.4 millivolt = 0.5 millivolt).

TMP14 pp. 138–139

36. E) The contraction of the ventricles lasts almost from the beginning of

the Q wave and continues to the end of the T wave. This interval is

called the Q-T interval and ordinarily lasts about 0.35 second.

TMP14 p. 137

37. A) By convention, the right arm is the negative electrode for lead II of

an ECG.

TMP14 p. 139

38. B) The augmented unipolar leads are obtained by connecting two

terminals to negative and one to positive. In case of aVF, the positive

terminal is connected to the left leg and the negative ones to the right

and left arms.

TMP14 pp. 140–141

39. D) Einthoven’s law states that the voltage in lead I plus the voltage in

lead III is equal to the voltage in lead II, which in this case is 2.0

millivolts.

TMP14 pp. 138–139

40. A) Chest leads (precordial leads, V1 to V6) are very sensitive to

electrical potential changes underneath the electrode and show the

electrical activity of the heart from base to apex. ST (or J point) elevation

or depression in precordial leads allows to identify the anterior or

posterior origin of the current of injury. ST (or J point) elevation in

bipolar and/or augmented unipolar leads help to refine the area of

ischemia (e.g., lateral, inferior).

TMP14 pp. 138–141

41. B) The mean electrical axis can be determined by ploing the resultant

voltage of the QRS for leads I, II, and III. The result is shown below and

has a value of −50 degrees.

TMP14 pp. 143–146

42. B) The heart rate can be calculated by 60 divided by the R-R interval,

which is 0.68 second. This calculation results in a heart rate of 88

beats/min.

TMP14 p. 137

43. B) In the figure, the QRS width is greater than 0.12 second, which

indicates a bundle branch block. Right bundle branch block is not a

listed answer. The correct answer is therefore left bundle branch block.

TMP14 pp. 149–150

44. C) Aortic stenosis induces left ventricular hypertrophy. Consequently,

patients often show a significant left axis deviation in their ECGs. A

third-degree A-V block will most likely not lead to any axis deviation.

The other answers associate with right axis deviation.

TMP14 pp. 148–150

45. D) Lead II has a positive vector at the 60-degree angle. The negative

end of lead II is at −120 degrees.

TMP14 pp. 143–146

46. D) Heart rate is calculated by 60/R-R interval and is 103 beats/min.

TMP14 p. 137

47. C) A high-voltage ECG (sum of voltages of leads I to III greater than 4

mV) is frequently observed in trained athletes driven by exerciseinduced cardiac hypertrophy. Infarcts and pericardial or pleural

effusion may result in lower voltage.

TMP14 pp. 150–151

48. B) The patient has a mean electrical axis of 60 degrees because of the

large deflection in lead II and zero in lead aVL. The axis of aVL is −30

degrees, which is perpendicular to lead II, and this indicates that the

axis must be 60 degrees.

TMP14 pp. 143–146

49. A) This patient has an acute anterior infarction in the left ventricle of

the heart. This diagnosis can be determined by ploing the currents of

injury from the different leads (see figure on the next page). The limb

leads are used to determine whether the infarction is coming from the

left or right side of the ventricle and from the base or inferior part of the

ventricle. The chest leads are used to determine whether it is an anterior

or posterior infarct. When we analyze the currents of injury, a negative

potential, caused by the current of injury, occurs in lead I and a positive

potential, caused by the current of injury, occurs in lead III. This is

determined by subtracting the J point from the TP segment. The

y g p g

negative end of the resultant vector originates in the ischemic area,

which is therefore the left side of the heart. In lead V2

, the chest lead, the

electrode is in a field of very negative potential, which occurs in patients

with an anterior lesion.

TMP14 pp. 154–155


50. D) The QRS wave ploed on lead I was −1.2 millivolts, and lead II was

+1.2 millivolts, so the absolute value of the deflections was the same.

Therefore, the mean electrical axis must be exactly halfway in between

these two leads, which is halfway between the lead II axis of 60 degrees

and the lead I negative axis of 180 degrees, which provides a value of

120 degrees.

TMP14 pp. 143–146

51. E) During the T-P interval in a patient with a damaged ventricle, the

only area depolarized is the damaged muscle. Therefore, the remainder

of the ventricle is repolarized. At the J point, the entire ventricle is

depolarized in a patient with a damaged cardiac muscle or in a patient

with a normal cardiac muscle. The area of the heart that is damaged will

not repolarize but remains depolarized at all times.

TMP14 pp. 152–154

52. A) Note in the figure in the next column that the QRS complex has a

positive deflection in lead I and a negative in lead III, which indicates

that there is a leftward axis deviation, which occurs during chronic

systemic hypertension. Pulmonary hypertension increases the

ventricular mass on the right side of the heart, which gives a right axis

deviation.

TMP14 p. 149

53. A) Because the deflection in this ECG is 0 in lead I, the axis has to be 90

degrees away from this lead. Therefore, the mean electrical axis must be

+90 degrees or −90 degrees. Because the aVF lead has a positive

deflection, the mean electrical axis must be at +90 degrees.

TMP14 pp. 143–146

54. B) The ECG from this patient has a positive deflection in aVF and a

negative deflection in standard limb lead I. Therefore, the mean

electrical axis is between 90 degrees and 180 degrees, which is a

rightward shift in the ECG mean electrical axis. Systemic hypertension,

aortic valve stenosis, and aortic valve regurgitation cause hypertrophy

of the left ventricle and thus a leftward shift in the mean electrical axis.

Pulmonary hypertension causes a rightward shift in the axis and is

therefore characterized by this ECG.

TMP14 pp. 149–150

55. D) A QRS axis of 120 degrees indicates a rightward shift. Because the

QRS complex is 0.18 second, this indicates a conduction block.

Therefore, the diagnosis that fits with these characteristics is a right

bundle branch block.

TMP14 pp. 149–150

56. D) In the figure on the next page, the current of injury is ploed at the

boom of the graph. This is not a plot of the QRS voltages but the

current of injury voltages. They are ploed for leads II and III, which are

both negative, and the resultant vector is nearly vertical. The negative

end of the vector points to where the current of injury originated, which

is in the apex of the ventricle. The elevation of the TP segment above the

J point indicates a posterior lesion. Therefore, the ECG is consistent with

acute posterior infarction in the apex of the ventricle.

TMP14 pp. 152–155

57. E) This patient has a difference in the atrial rate of 100 and in the

ventricular rate of 25. The 25 rate in the ventricles is indicative of a

rhythm starting in the Purkinje fibers. A-V block is occurring, but it

comes and goes, which is only fulfilled by Stokes-Adams syndrome.

TMP14 pp. 158–160

58. C) A person with atrial fibrillation has a rapid, irregular heart rate. The

P waves are missing or are very weak. The atria exhibit circus

movements, and atrial volume is often increased, causing the atrial

fibrillation.

TMP14 pp. 166–167

59. A) Circus movements occur in ventricular muscle, particularly in

persons with a dilated heart or decreases in conduction velocity. High

extracellular potassium and sympathetic stimulation, not

parasympathetic stimulation, increase the tendency for circus

movements. A longer refractory period tends to prevent circus

movements of the heart because when the impulses travel around the

heart and contact the area of ventricular muscle that has a longer

refractory period, the action potential stops at this point.

TMP14 pp. 163–165

60. D) When a patient has no P waves and a low heart rate, it is likely that

the impulse leaving the sinus node is totally blocked before entering the

atrial muscle, which is called sinoatrial block. The ventricles pick up the

new rhythm, usually initiated in the A-V node at this point, which

results in a heart rate of 40 to 60/min. In contrast, during sinus

bradycardia, P waves are still associated with each QRS complex. In

first-, second-, and third-degree heart block, P waves are present in each

of these instances, although some are not associated with QRS complex.

TMP14 p. 158

61. B) By definition, first-degree A-V heart block occurs when the P-R

interval exceeds a value of 0.20 second but without any dropped QRS

waves. This ECG shows first-degree block. In this figure, the P-R interval

is about 0.30 second, which is considerably prolonged. However, there

are no dropped QRS waves. During second-degree A-V block, QRS

waves are dropped.

TMP14 pp. 158–159

62. E) In the figure below, note that the premature ventricular contractions

(PVCs) have a wide and tall QRS wave in the ECG. The mean electrical

axis of the premature contraction can be determined by ploing these

large QRS complexes on the standard limb leads. The PVC originates at

the negative end of the resultant mean electrical axis, which is at the

base of the ventricle. Notice that the QRS of the PVC is wider and much

taller than the normal QRS waves in this ECG.

TMP14 p. 161

63. B) This patient has atrial fluer, which is characterized by several P

waves for each QRS complex. This ECG has two P waves for every QRS.

Notice the rapid heart rate, which is characteristic of atrial fluer.

TMP14 p. 167

64. E) The average ventricular rate is 150 beats/min in this ECG, which is

typical of atrial fluer. Again notice that the heart rate is irregular

because of the inability of the impulses to quickly pass through the A-V

node because of its refractory period.

TMP14 p. 167

65. C) A decreased velocity of conduction is a major trigger for reentry/circus movements and generation of ventricular fibrillation. The

circus movements are multiple and generate in the ventricle, not from

the atria, and travel to the ventricles. A shorter refractory period

facilitates circus movements. Ventricular fibrillation is a medical

emergency, always requires intervention, and does not reverse

spontaneously.

TMP14 pp. 163–165

66. D) An inverted P wave occurs in patients with a premature contraction

originating in the A-V junction. If the P wave occurs after the QRS

complex, the junctional contraction started low in the A-V junction.

Junctional contractions originating high in the A-V junction will have a

P wave that occurs before the QRS, and likewise one originating in the

middle of junction occurs during the QRS.

TMP14 pp. 160–161

67. E) The term “paroxysmal” means that the heart rate becomes rapid in

paroxysms, with the paroxysm beginning suddenly and lasting for a

few seconds, a few minutes, a few hours, or much longer. Then the

paroxysm usually ends as suddenly as it began, and the pacemaker

shifts back to the S-A node. The mechanism by which this phenomenon

is believed to occur is by a re-entrant circus movement feedback

pathway that sets up an area of local repeated self–re-excitation. The

ECG shown is ventricular paroxysmal tachycardia. That the origin is in

the ventricles can be determined because of the changes in the QRS

complex, which have high voltages and look much different than the

preceding normal QRS complexes. This is very characteristic of a

ventricular irritable locus.

TMP14 pp. 162–163

68. C) Notice in this ECG that a P wave precedes each of the first four QRS

complexes. After that we see a P wave but a dropped QRS wave, which

is characteristic of second-degree A-V block.

TMP14 pp. 158–159

69. A) A person with atrial fibrillation has a rapid, irregular heart rate. The

P waves are missing or are very weak. The atria exhibit circus

movements and often are very enlarged, causing the atrial fibrillation.

TMP14 pp. 166–167

70. B) Ventricular fibrillation often occurs in a heart exposed to a 60-cycle

alternating current. An increased conduction velocity through the heart

muscle or a shortened conduction pathway around the heart decreases

the probability of re-entrant pathways. A shortened ventricular

refractory period increases the possibility of fibrillation. Thus, when the

electrical stimulus travels around the heart and reaches the ventricular

muscle that was again initially stimulated, the risk of ventricular

fibrillation increases because the muscle will be out of the refractory

period.

TMP14 pp. 164–165

71. C) During a Stokes-Adams syndrome aack, total A-V block suddenly

begins, and the duration of the block may be a few seconds or even

several weeks. The new pacemaker of the heart is distal to the point of

blockade, usually in some part of the Purkinje fibers or the A-V bundle.

TMP14 pp. 159–160

72. D) One of the early ECG signs of cardiac ischemia is the development

of repolarization abnormalities, reflected by changes in the T wave.

These changes may occur without changes in J point-ST segments.

Septum and endocardium depolarize first and repolarize last.

Repolarized areas will have + charge and describe a + vector toward the

apex. The first area to repolarize is toward the apex. High potassium

may result in peaked T waves and a prolonged PR.

TMP14 p. 156

73. A) The relationship between the P waves and the QRS complexes

appears to be normal, and there are no missing beats. Therefore, this

patient has a sinus rhythm, and there is no heart block. There is also no

ST-segment depression in this patient. Because we have normal P and

QRS and T waves, this condition is sinus tachycardia.

TMP14 p. 157

74. A) The heartbeat immediately following a premature atrial contraction

weakens because the diastolic period is very short in this condition.

Therefore, the ventricular filling time is very short, and thus the stroke

volume decreases. The P wave is usually visible in this arrhythmia

unless it coincides with the QRS complex. The probability of these

premature contractions increases in people with toxic irritation of the

heart and local ischemic areas.

TMP14 p. 160

75. C) During atrial paroxysmal tachycardia, the impulse is initiated by an

ectopic focus somewhere in the atria. If the point of initiation is near the

A-V node, the P wave travels backward toward the S-A node and then

forward into the ventricles at the same time. Therefore, the P wave will

be inverted.

TMP14 pp. 160–161

76. A) This ECG has characteristics of atrial paroxysmal tachycardia, which

means that the tachycardia may come and go at random times. The basic

shape of the QRS complex and its magnitude are virtually unchanged

from the normal QRS complexes, which eliminates the possibility of

ventricular paroxysmal tachycardia. This ECG is not characteristic of

atrial fluer because there is only one P wave for each QRS complex.

TMP14 162–163

77. C) In PVCs, QRS is prolonged because impulse is conducted through

muscle, which has slow conduction, and QRS voltage is high because

one side depolarizes ahead of the other. Consequently, the T wave is

inverted because slow conduction causes the area to first depolarize to

also repolarize first (opposite of normal). P waves are absent, PVCs can

develop in individuals with healthy heart but also in pathological

conditions, and the impulse does not travel backwards to the atria and

then forward.

TMP14 p. 161

78. A) In rare instances, impulses from the S-A node are blocked. This

causes cessation of P waves and a new pacemaker assuming the

generation and conduction of the impulse. Usually, the next region of

heart with the fastest discharge rate is the A-V node. B to C are features

of A-V blocks. A full compensatory pause is a feature of PVCs.

TMP14 p. 158

79. E) The absence of P wave and fully irregular heart rate suggest atrial

fibrillation and not fluer (sawtooth paern, heart rate is not fully

irregular), atrial tachycardia (P waves are present, may show altered

shape, heart rate is regular), or first-degree A-V block (P waves are

present, prolonged PR). The left axis deviation and QRS within normal

limits (upper limit) suggest left ventricular hypertrophy, in the context

of a “history of hypertension.” There is no left or right bundle branch

block because the QRS is normal in duration.

TMP14 pp. 166–167


Unit IV: The Circulation

1. Listed below are the hydrostatic and oncotic pressures within a

microcirculatory bed.

Plasma colloid osmotic pressure = 25 mm Hg

Capillary hydrostatic pressure = 25 mm Hg

Venous hydrostatic pressure = 5 mm Hg

Arterial pressure = 80 mm Hg

Interstitial fluid hydrostatic pressure = −5 mm Hg

Interstitial colloid osmotic pressure = 10 mm Hg

Capillary filtration rate = 150 ml/min

What is the capillary filtration coefficient (in ml/min/mm Hg) for this

capillary wall?

A) 5

B) 10

C) 15

D) 20

E) 25

2. A healthy 60-year-old woman with a 10-year history of

hypertension stands up from a supine position. Which set of

cardiovascular changes is most likely to occur in response to

standing up from a supine position?

Sympathetic Nerve Activity Cardiac Contractility Heart Rate

A) ↑ ↑ ↑

B) ↑ ↑ ↓

C) ↑ ↓ ↓

D) ↑ ↓ ↑

E) ↓ ↓ ↓

F) ↓ ↓ ↑

G) ↓ ↑ ↑

H) ↓ ↑ ↓

3. In an experimental study, administration of a drug decreases the

diameter of arterioles in the muscle bed of an animal subject.

Which set of physiological changes would be expected to occur in

response to the decrease in diameter?

Vascular Conductance Capillary Filtration Blood Flow

A) ↑ ↑ ↑

B) ↑ ↓ ↑

C) ↑ ↓ ↓

D) ↑ ↑ ↓

E) ↓ ↓ ↓

F) ↓ ↑ ↓

G) ↓ ↑ ↑

H) ↓ ↓ ↑

4. A 60-year-old woman has experienced dizziness for the past 6

months when geing out of bed in the morning and when standing

up. Her mean arterial pressure is 130/90 mm Hg while lying down

and 95/60 while siing. Which set of physiological changes would

be expected in response to moving from a supine to an upright

position?

Parasympathetic Nerve Activity Plasma Renin Activity Sympathetic Activity

A) ↑ ↑ ↑

B) ↑ ↓ ↑

C) ↑ ↓ ↓

D) ↑ ↑ ↓

E) ↓ ↓ ↓

F) ↓ ↑ ↓

G) ↓ ↑ ↑

H) ↓ ↓ ↑

5. A 35-year-old woman visits her family practitioner for an

examination. She has a blood pressure of 160/75 mm Hg and a

heart rate of 74 beats/min. Further tests by a cardiologist reveal that

the patient has moderate aortic regurgitation. Which set of changes

would be expected in this patient?

Pulse Pressure Systolic Pressure Stroke Volume

A) ↑ ↑ ↑

B) ↑ ↓ ↑

C) ↑ ↓ ↓

D) ↑ ↑ ↓

E) ↓ ↓ ↓

F) ↓ ↑ ↓

G) ↓ ↑ ↑

H) ↓ ↓ ↑

6. A healthy 27-year-old female medical student runs a 5K race.

Which set of physiological changes is most likely to occur in this

woman’s skeletal muscles during the race?

Arteriole Resistance Tissue pH Tissue Carbon Dioxide Concentration

A) ↑ ↑ ↑

B) ↑ ↑ ↓

C) ↑ ↓ ↓

D) ↑ ↓ ↑

E) ↓ ↓ ↓

F) ↓ ↓ ↑

G) ↓ ↑ ↑

H) ↓ ↑ ↓

7. Cognitive stimuli such as reading, problem solving, and talking all

result in significant increases in cerebral blood flow. Which set of

changes in cerebral tissue concentrations is the most likely

explanation for the increase in cerebral blood flow?

Carbon Dioxide pH Adenosine

A) ↑ ↑ ↑

B) ↑ ↓ ↑

C) ↑ ↓ ↓

D) ↑ ↑ ↓

E) ↓ ↓ ↓

F) ↓ ↑ ↓

G) ↓ ↑ ↑

H) ↓ ↓ ↑

8. Histamine is infused into the brachial artery. Which set of

microcirculatory changes would be expected in the infused arm?

Capillary Water Permeability Capillary Hydrostatic Pressure Interstitial Hydrostatic Pressure

A) ↑ ↑ ↑

B) ↑ ↑ ↓

C) ↑ ↓ ↓

D) ↑ ↓ ↑

E) ↓ ↓ ↓

F) ↓ ↓ ↑

G) ↓ ↑ ↑

H) ↓ ↑ ↓

9. An increase in shear stress in a blood vessel results in which

change?

A) Decreased endothelin production

B) Decreased cyclic guanosine monophosphate production

C) Increased nitric oxide release

D) Increased renin production

E) Decreased prostacyclin production

10. A 65-year-old man with a 10-year history of essential hypertension

is being treated with an angiotensin-converting enzyme (ACE)

inhibitor. Which set of changes would be expected to occur in

response to the ACE inhibitor drug therapy?

Plasma Renin Concentration Total Peripheral Resistance Blood Pressure

A) ↑ ↑ ↑

B) ↑ ↑ ↓

C) ↑ ↓ ↓

D) ↑ ↓ ↑

E) ↓ ↓ ↓

F) ↓ ↓ ↑

G) ↓ ↑ ↑

H) ↓ ↑ ↓

11. The diameter of a precapillary arteriole is decreased in a muscle

vascular bed. An increase in which of the following would be

expected?

A) Capillary filtration rate

B) Vascular conductance

C) Capillary blood flow

D) Capillary hydrostatic pressure

E) Arteriolar resistance

12. A 55-year-old man with a history of normal health visits his

physician for a checkup. The physical examination reveals that his

blood pressure is 170/98 mm Hg. Further tests indicate that he has

renovascular hypertension as a result of stenosis in the left kidney.

Which set of findings would be expected in this man with

renovascular hypertension?

Total Peripheral Resistance Plasma Renin Activity Plasma Aldosterone Concentration

A) ↑ ↑ ↑

B) ↑ ↓ ↑

C) ↑ ↓ ↓

D) ↑ ↑ ↓

E) ↓ ↓ ↓

F) ↓ ↑ ↓

G) ↓ ↑ ↑

H) ↓ ↓ ↑

13. Under control conditions, flow through a blood vessel is 100

ml/min with a pressure gradient of 50 mm Hg. What would be the

approximate flow through the vessel after increasing the vessel

diameter by 100%, assuming that the pressure gradient is

maintained at 50 mm Hg?

A) 200 ml/min

B) 400 ml/min

C) 800 ml/min

D) 1600 ml/min

E) 700 ml/min

14. A 24-year-old woman delivers a 6-lb, 8-oz baby girl. The newborn

is diagnosed as having patent ductus arteriosus. Which set of

changes would be expected in this baby?

Pulse Pressure Stroke Volume Systolic Pressure

A) ↑ ↑ ↑

B) ↑ ↓ ↑

C) ↑ ↓ ↓

D) ↑ ↑ ↓

E) ↓ ↓ ↓

F) ↓ ↑ ↓

G) ↓ ↑ ↑

H) ↓ ↓ ↑

15. A 72-year-old man had surgery to remove an abdominal tumor.

Pathohistological studies revealed that the tumor mass contained a

large number of vessels. The most likely stimulus for the growth of

vessels in a solid tumor is a decrease in which of the following?

A) Growth hormone

B) Plasma glucose concentration

C) Angiostatin growth factor

D) Vascular endothelial growth factor

E) Tissue oxygen concentration

16. Which set of changes would be expected to cause the greatest

increase in the net movement of sodium across a muscle capillary

wall?

Wall Permeability to Sodium Wall Surface Area Concentration Difference Across Wall

A) ↑ ↑ ↑

B) ↑ ↑ ↓

C) ↑ ↓ ↓

D) ↑ ↓ ↑

E) ↓ ↓ ↓

F) ↓ ↓ ↑

G) ↓ ↑ ↑

H) ↓ ↑ ↓

17. While participating in a cardiovascular physiology laboratory, a

medical student isolates an animal’s carotid artery proximal to the

carotid bifurcation and partially constricts the artery with a tie

around the vessel. Which set of changes would be expected to

occur in response to constriction of the carotid artery?

Heart Rate Parasympathetic Nerve Activity Total Peripheral Resistance

A) ↑ ↑ ↑

B) ↑ ↑ ↓

C) ↑ ↓ ↓

D) ↑ ↓ ↑

E) ↓ ↓ ↓

F) ↓ ↓ ↑

G) ↓ ↑ ↑

H) ↓ ↑ ↓

18. A 35-year-old woman visits her family practice physician for an

examination. She has a mean arterial blood pressure of 105 mm Hg

and a heart rate of 74 beats/min. Further tests by a cardiologist

reveal that the patient has moderate aortic valve stenosis. Which

set of changes would be expected in this patient?

Pulse Pressure Stroke Volume Systolic Pressure

A) ↑ ↑ ↑

B) ↑ ↓ ↑

C) ↑ ↓ ↓

D) ↑ ↑ ↓

E) ↓ ↓ ↓

F) ↓ ↑ ↓

G) ↓ ↑ ↑

H) ↓ ↓ ↑

19. A 60-year-old man visits his family practitioner for an annual

examination. He has a mean blood pressure of 130 mm Hg and a

heart rate of 78 beats/min. His plasma cholesterol level is in the

upper 25th percentile, and he is diagnosed as having

atherosclerosis. Which set of changes would be expected in this

patient?

Pulse Pressure Arterial Compliance Systolic Pressure

A) ↑ ↑ ↑

B) ↑ ↓ ↑

C) ↑ ↓ ↓

D) ↑ ↑ ↓

E) ↓ ↓ ↓

F) ↓ ↑ ↓

G) ↓ ↑ ↑

H) ↓ ↓ ↑

20. While participating in a cardiovascular physiology laboratory, a

medical student isolates the carotid artery of an animal and

partially constricts the artery with a tie around the vessel. Which

set of changes would be expected to occur in response to

constriction of the carotid artery?

Sympathetic Nerve Activity Renal Blood Flow Total Peripheral Resistance

A) ↑ ↑ ↑

B) ↑ ↓ ↑

C) ↑ ↓ ↓

D) ↑ ↑ ↓

E) ↓ ↓ ↓

F) ↓ ↑ ↓

G) ↓ ↑ ↑

H) ↓ ↓ ↑

21. Which one of the following would tend to increase capillary

filtration rate?

A) Decreased capillary hydrostatic pressure

B) Decreased plasma colloid osmotic pressure

C) Decreased interstitial colloid osmotic pressure

D) Decreased capillary water permeability

E) Increased arteriolar resistance

22. A 72-year-old man had surgery to remove an abdominal tumor.

Findings of pathohistological studies reveal that the tumor mass

contains a large number of blood vessels. The most likely stimulus

for the growth of vessels in a solid tumor is an increase in which of

the following?

A) Growth hormone

B) Plasma glucose concentration

C) Angiostatin growth factor

D) Tissue oxygen concentration

E) Vascular endothelial growth factor (VEGF)

23. The diameter of a precapillary arteriole is decreased in a muscle

vascular bed. Which change in the microcirculation would be

expected?

A) Decreased capillary filtration rate

B) Increased interstitial volume

C) Increased lymph flow

D) Increased capillary hydrostatic pressure

E) Decreased arteriolar resistance

24. A 50-year-old man has a 3-year history of hypertension. He

reports fatigue and occasional muscle cramps. There is no family

history of hypertension. The patient has not had any other

significant medical problems in the past. Examination reveals a

blood pressure of 168/104 mm Hg. Additional laboratory tests

indicate that the patient has primary hyperaldosteronism. Which

set of findings would be expected in this man with primary

hyperaldosteronism hypertension?

Extracellular Fluid Volume Plasma Renin Activity Plasma Potassium Concentration

A) ↑ ↑ ↑

B) ↑ ↓ ↑

C) ↑ ↓ ↓

D) ↑ ↑ ↓

E) ↓ ↓ ↓

F) ↓ ↑ ↓

G) ↓ ↑ ↑

H) ↓ ↓ ↑

25. A decrease in which of the following would tend to increase

lymph flow?

A) Hydraulic conductivity of the capillary wall

B) Plasma colloid osmotic pressure

C) Capillary hydrostatic pressure

D) Vascular conductance

E) B and D

26. In control conditions, flow through a blood vessel is 100 ml/min

under a pressure gradient of 50 mm Hg. What would be the

approximate flow through the vessel after increasing the vessel

diameter to four times normal, assuming that the pressure gradient

was maintained at 50 mm Hg?

A) 300 ml/min

B) 1600 ml/min

C) 1000 ml/min

D) 16,000 ml/min

E) 25,600 ml/min

27. A 50-year-old woman has a renal plasma flow of 600 ml/min and

hematocrit of 50. Her arterial pressure is 125 mm Hg and renal

venous pressure is 5 mm Hg. What is the total renal vascular

resistance (in mm Hg/ml/min) in this woman?

A) 0.05

B) 0.10

C) 0.50

D) 1.00

E) 1.50

28. An increase in which of the following would be expected to

decrease blood flow in a vessel?

A) Pressure gradient across the vessel

B) Radius of the vessel

C) Plasma colloid osmotic pressure

D) Viscosity of the blood

E) Plasma sodium concentration

29. Assuming that vessels A to D are the same length, which one has

the greatest flow?

Pressure Gradient Radius Viscosity

A) 100 1 10

B) 50 2 5

C) 25 4 2

D) 10 6 1

30. A 22-year-old man enters the hospital emergency department after

severing a major artery in a motorcycle accident. It is estimated

that he has lost approximately 700 ml of blood. His blood pressure

is 90/55 mm Hg. Which set of changes would be expected in

response to hemorrhage in this man?

Heart Rate Parasympathetic Nerve Activity Plasma Renin Activity

A) ↑ ↑ ↑

B) ↑ ↓ ↑

C) ↑ ↓ ↓

D) ↑ ↑ ↓

E) ↓ ↓ ↓

F) ↓ ↑ ↓

G) ↓ ↑ ↑

H) ↓ ↓ ↑

31. A healthy 28-year-old woman stands up from a supine position.

Moving from a supine to a standing position results in a transient

decrease in arterial pressure that is detected by arterial

baroreceptors located in the aortic arch and carotid sinuses. Which

set of cardiovascular changes is most likely to occur in response to

activation of the baroreceptors?

Mean Circulatory Filling Pressure Strength of Cardiac Contraction Sympathetic Nerve Activity

A) ↑ ↑ ↑

B) ↑ ↓ ↑

C) ↑ ↓ ↓

D) ↑ ↑ ↓

E) ↓ ↓ ↓

F) ↓ ↑ ↓

G) ↓ ↑ ↑

H) ↓ ↓ ↑

32. An ACE inhibitor is administered to a 65-year-old man with a 20-

year history of hypertension. The drug lowered his arterial

pressure and increased his plasma levels of renin. Which

mechanism would best explain the decrease in arterial pressure?

A) Inhibition of angiotensin I

B) Decreased conversion of angiotensinogen toangiotensin I

C) Decreased plasma levels of bradykinin

D) Decreased plasma levels of prostacyclin

E) Decreased formation of angiotensin II

33. A 25-year-old man enters the hospital emergency department after

severing a major artery during a farm accident. It is estimated that

the patient has lost approximately 800 ml of blood. His mean blood

pressure is 65 mm Hg, and his heart rate is elevated as a result of

activation of the chemoreceptor reflex. Which set of changes in

plasma concentration would be expected to cause the greatest

activation of the chemoreceptor reflex?

Oxygen Carbon Dioxide Hydrogen

A) ↑ ↑ ↑

B) ↑ ↓ ↑

C) ↑ ↓ ↓

D) ↑ ↑ ↓

E) ↓ ↓ ↓

F) ↓ ↑ ↓

G) ↓ ↑ ↑

H) ↓ ↓ ↑

34. Under normal physiological conditions, blood flow to the skeletal

muscles is determined mainly by which of the following?

A) Sympathetic nerves

B) Angiotensin II

C) Vasopressin

D) Local metabolic factors

E) Capillary osmotic pressure

35. A healthy 22-year-old female medical student has an exercise

stress test at a local health club. An increase in which of the

following is most likely to occur in this woman’s skeletal muscles

during exercise?

A) Vascular conductance

B) Blood flow

C) Carbon dioxide concentration

D) Arteriolar diameter

E) All the above

36. Which of the following segments of the circulatory system has the

lowest velocity of blood flow?

A) Aorta

B) Arteries

C) Capillaries

D) Veins

37. Listed below are the hydrostatic and oncotic pressures within a

microcirculatory bed.

Plasma colloid osmotic pressure = 25 mm Hg

Capillary hydrostatic pressure = 25 mm Hg

Venous hydrostatic pressure = 5 mm Hg

Arterial pressure = 80 mm Hg

Interstitial hydrostatic pressure = −5 mm Hg

Interstitial colloid osmotic pressure = 5 mm Hg

Filtration coefficient = 15 ml/min/mm Hg

What is the filtration rate (ml/min) of the capillary wall?

A) 100

B) 150

C) 200

D) 250

E) 300

38. Which blood vessel has the highest vascular resistance?

Blood Flow (ml/min) Pressure Gradient (mm Hg)

A) 1000 100

B) 1200 60

C) 1400 20

D) 1600 80

E) 1800 40

39. A 2-fold increase in which of the following would result in the

greatest increase in the transport of oxygen across the capillary

wall?

A) Capillary hydrostatic pressure

B) Intercellular clefts in the capillary wall

C) Oxygen concentration gradient

D) Plasma colloid osmotic pressure

E) Capillary wall hydraulic permeability

40. A balloon catheter is advanced from the superior vena cava into

the heart and inflated to increase atrial pressure by 5 mm Hg.

Which of the following would be expected to occur in response to

the elevated atrial pressure?

A) Decreased atrial natriuretic peptide

B) Increased angiotensin II

C) Increased aldosterone

D) Decreased renal sympathetic nerve activity

41. Which of the following vessels has the greatest total crosssectional area in the circulatory system?

A) Aorta

B) Small arteries

C) Capillaries

D) Venules

E) Vena cava

42. An increase in atrial pressure results in which of the following?

A) Increased plasma atrial natriuretic peptide

B) Increase in plasma angiotensin II concentration

C) Decrease in plasma aldosterone concentration

D) Decrease in sodium excretion

E) A and C

43. Autoregulation of tissue blood flow in response to an increase in

arterial pressure occurs as a result of which of the following?

A) Decrease in vascular resistance

B) Initial decrease in vascular wall tension

C) Excess delivery of nutrients such as oxygen to the tissues

D) Decrease in tissue metabolism

44. Which component of the circulatory system contains the largest

percentage of the total blood volume?

A) Arteries

B) Capillaries

C) Veins

D) Pulmonary circulation

E) Heart

45. Which set of changes would be expected to occur 2 weeks after a

50% reduction in renal artery pressure?

Plasma Renin Plasma Aldosterone Concentration Glomerular Filtration Rate

A) ↑ ↑ ↑

B) ↑ ↑ ↓

C) ↑ ↓ ↓

D) ↑ ↓ ↑

E) ↓ ↓ ↓

F) ↓ ↓ ↑

G) ↓ ↑ ↑

H) ↓ ↑ ↓

46. An increase in which of the following tends to decrease capillary

filtration rate?

A) Capillary hydrostatic pressure

B) Plasma colloid osmotic pressure

C) Interstitial colloid osmotic pressure

D) Venous hydrostatic pressure

E) Arteriolar diameter

47. A decrease in which of the following would be expected to occur

in a person 2 weeks after an increase in sodium intake?

A) Angiotensin II

B) Sodium Excretion

C) Aldosterone

D) Atrial natriuretic peptide

E) A and C

48. Which of the following would tend to increase lymph flow?

A) Increase capillary hydrostatic pressure

B) Increased plasma colloid osmotic pressure

C) Increased interstitial volume

D) Decreased arteriolar diameter

E) A and C

49. An increase in the production of which of the following would

most likely result in chronic hypertension?

A) Aldosterone

B) Prostacyclin

C) Angiotensin II

D) Nitric oxide

E) A and C

50. Which of the following capillaries has the highest capillary

permeability to plasma albumin?

A) Glomerular

B) Liver

C) Muscle

D) Intestinal

E) Brain

51. Which of the following would be expected to occur during a

Cushing reaction caused by brain ischemia?

A) Increase in parasympathetic activity

B) Decrease in arterial pressure

C) Decrease in heart rate

D) Increase in sympathetic activity

52. Which of the following tends to increase the net movement of

glucose across a capillary wall?

A) Increase in plasma sodium concentration

B) Increase in the concentration difference of glucose across the

wall

C) Decrease in wall permeability to glucose

D) Decrease in wall surface area without an increase in the

number of pores

E) Decrease in plasma potassium concentration

53. A 65-year-old man has congestive heart failure. He has a cardiac

output of 4 l/min, arterial pressure of 115/85 mm Hg, and heart rate

of 90 beats/min. Further tests by a cardiologist reveal that the

patient has a right atrial pressure of 10 mm Hg. An increase in

which of the following would be expected in this patient?

A) Plasma colloid osmotic pressure

B) Interstitial colloid osmotic pressure

C) Arterial pressure

D) Cardiac output

E) Vena cava hydrostatic pressure

54. Which set of changes would be expected to occur in response to a

direct increase in renal arterial pressure in kidneys without an

intact tubuloglomerular feedback system?

Glomerular Filtration Sodium Excretion Water Excretion Rate

A) ↑ ↑ ↑

B) ↑ ↑ ↓

C) ↑ ↓ ↓

D) ↑ ↓ ↑

E) ↓ ↓ ↓

F) ↓ ↓ ↑

G) ↓ ↑ ↑

H) ↓ ↑ ↓

55. Which part of the circulation has the highest compliance?

A) Capillaries

B) Large arteries

C) Veins

D) Aorta

E) Small arteries

56. An increase in which of the following tends to increase pulse

pressure?

A) Systolic pressure

B) Capillary hydrostatic pressure

C) Arterial compliance

D) Stroke volume

E) A and D

57. Which set of physiological changes would be expected to occur in

a person who stands up from a supine position?

Venous Hydrostatic Pressure in Legs Heart Rate Renal Blood Flow

A) ↑ ↑ ↑

B) ↑ ↑ ↓

C) ↑ ↓ ↓

D) ↓ ↓ ↓

E) ↓ ↓ ↑

F) ↓ ↑ ↑

58. Which one of the following compensatory physiological changes

would be expected to occur in a person who stands up from a

supine position?

A) Increased parasympathetic nerve activity

B) Increased sympathetic nerve activity

C) Decreased heart rate

D) Decreased heart contractiltiy

59. Blood flow to a tissue remains relatively constant despite a

reduction in arterial pressure (autoregulation). Which of the

following would be expected to occur in response to the increases

in arterial pressure?

A) Increased conductance

B) Increased tissue oxygen concentration

C) Decreased vascular resistance

D) Increased arteriolar diameter

60. Which of the following would have the slowest rate of net

movement across the capillary wall?

A) Sodium

B) Albumin

C) Glucose

D) Oxygen

61. An increase in which of the following tends to increase capillary

filtration rate?

A) Capillary wall hydraulic conductivity

B) Arteriolar resistance

C) Plasma colloid osmotic pressure

D) Interstitial hydrostatic pressure

E) Plasma sodium concentration

62. The tendency for turbulent flow is greatest in which of the

following?

A) Arterioles

B) Capillaries

C) Small arterioles

D) Aorta

63. A 60-year-old man has a mean arterial blood pressure of 130 mm

Hg, a heart rate of 78 beats/min, a right atrial pressure of 0 mm Hg,

and a cardiac output of 3.5 L/min. He also has a pulse pressure of

35 mm Hg and a hematocrit of 40. What is the approximate total

peripheral vascular resistance in this man?

A) 17 mm Hg/l/min

B) 1.3 mm Hg/l/min

g

C) 13 mm Hg/l/min

D) 27 mm Hg/l/min

E) 37 mm Hg/l/min

64. Which pressure is normally negative in a muscle capillary bed in

the lower extremities?

A) Plasma colloid osmotic pressure

B) Capillary hydrostatic pressure

C) Interstitial hydrostatic pressure

D) Interstitial colloid osmotic pressure

E) Venous hydrostatic pressure

65. What would tend to increase a person’s pulse pressure?

A) Decreased stroke volume

B) Increased arterial compliance

C) Hemorrhage

D) Patent ductus

E) Decreased venous return

66. Movement of solutes such as Na+

 across the capillary walls occurs

primarily by which process?

A) Filtration

B) Active transport

C) Vesicular transport

D) Diffusion

67. What would increase venous hydrostatic pressure in the legs?

A) Decrease in right atrial pressure

B) Pregnancy

C) Decreased movement of leg muscles

D) Abdominal compression of vena cava by a solid tumor in the

abdomen

E) B and D

68. A nitric oxide donor is infused into the brachial artery of a 22-

year-old man. Which set of microcirculatory changes would be

expected in the infused arm?

Capillary Hydrostatic Pressure Interstitial Hydrostatic Pressure Lymph Flow

A) ↑ ↑ ↑

B) ↑ ↑ ↓

C) ↑ ↓ ↓

D) ↑ ↓ ↑

E) ↓ ↓ ↓

F) ↓ ↓ ↑

G) ↓ ↑ ↑

H) ↓ ↑ ↓

69. What often occurs in decompensated heart failure?

A) Increased renal loss of sodium and water

B) Decreased mean systemic filling pressure

C) Increased norepinephrine in cardiac sympathetic nerves

D) Orthopnea

E) Weight loss

70. Which condition often occurs in progressive hemorrhagic shock?

A) Vasomotor center failure

B) Increased urine output

C) Tissue alkalosis

D) Decreased capillary permeability

E) Increased mean systemic filling pressure

71. A 50-year-old woman received an overdose of furosemide, and

her arterial pressure decreased to 70/40. Her heart rate is 120, and

her respiratory rate is 30/min. What therapy would you

recommend?

A) Whole blood infusion

B) Plasma infusion

C) Infusion of a balanced electrolyte solution

D) Infusion of a sympathomimetic drug

E) Administration of a glucocorticoid

72. A 30-year-old woman comes to a local emergency department

with severe vomiting. She has pale skin, tachycardia, an arterial

pressure of 70/45, and trouble walking. What therapy do you

recommend to prevent shock?

A) Infusion of packed red blood cells

B) Administration of an antihistamine

C) Infusion of a balanced electrolyte solution

D) Infusion of a sympathomimetic drug

E) Administration of a glucocorticoid

Modified from Guyton AC, Jones CE, Coleman TB: Circulatory Physiology:

Cardiac Output and Its Regulation, 2nd ed. Philadelphia: WB Saunders,

1973.

73. In the above figure, for the cardiac output and venous return

curves defined by the solid red lines (with the equilibrium at A),

which of the following options is true?

A) Mean systemic filling pressure is 12 mm Hg

B) Right atrial pressure is 2 mm Hg

C) Resistance to venous return is 1.4 mm Hg/l/min

D) Pulmonary arterial flow is approximately 7 l/min

E) Resistance to venous return is 0.71 mm Hg/l/min

74. A 30-year-old man is resting, and his sympathetic output increases

to maximal values. Which set of changes would be expected in

response to this increased sympathetic output?

Resistance to Venous Return Mean Systemic Filling Pressure Venous Return

A) ↑ ↑ ↑

B) ↑ ↓ ↑

C) ↑ ↓ ↓

D) ↑ ↑ ↓

E) ↓ ↓ ↓

F) ↓ ↑ ↓

G) ↓ ↑ ↑

H) ↓ ↓ ↑

75. If a patient has an oxygen consumption of 240 ml/min, a

pulmonary vein oxygen concentration of 180 ml/l of blood, and a

pulmonary artery oxygen concentration of 160 ml/l of blood units,

what is the cardiac output in l/min?

A) 8

B) 10

C) 12

D) 16

E) 20

76. What normally causes the cardiac output curve to shift to the right

along the right atrial pressure axis?

A) Changing intrapleural pressure to −1 mm Hg

B) Increasing mean systemic filling pressure

C) Taking a patient off a mechanical ventilator and allowing

normal respiration

D) Decreasing intrapleural pressure to −7 mm Hg

E) Breathing against a negative pressure

77. What normally causes the cardiac output curve to shift to the left

along the right atrial pressure axis?

A) Surgically opening the chest

B) Severe cardiac tamponade

C) Breathing against a negative pressure

D) Playing a trumpet

E) Positive-pressure breathing

78. What will elevate the plateau of the cardiac output curve?

A) Surgically opening the thoracic cage

B) Connecting a patient to a mechanical ventilator

C) Cardiac tamponade

D) Increasing parasympathetic stimulation of the heart

E) Increasing sympathetic stimulation of the heart

79. What is normally associated with an increased cardiac output?

A) Increased parasympathetic stimulation

B) Atrioventricular (A-V) fistula

C) Decreased blood volume

D) Polycythemia

E) Severe aortic regurgitation

80. Which condition would be expected to decrease mean systemic

filling pressure?

A) Norepinephrine administration

B) Increased blood volume

C) Increased sympathetic stimulation

D) Increased venous compliance

E) Skeletal muscle contraction

81. A 35-year-old man undergoes several cardiac test during exercise.

The following measurements are made:

Right atrial pressure

= +2 mm Hg

Left atrial pressure

= +7 mm Hg

Left ventricular end diastolic pressure

= +10 mm Hg

Mean systemic filling pressure

= +12 mm Hg

Cardiac output = 10 l/min

What is the resistance to venous return (mm Hg/l/min) in this

individual?

A) 0.1

B) 0 .5

C) 1.0

D) 1.4

E) 2.0

82. In which condition would you expect a decreased resistance to

venous return?

A) Anemia

B) Increased venous resistance

C) Increased arteriolar resistance

D) Increased sympathetic output

E) Obstruction of veins

83. Which of the following would decrease cardiac output?

A) Increased stroke volume

B) Increased heart rate

C) Increased mean systemic filling pressure

D) Increased resistance to venous return

E) Increased venous return

84. In which condition would you normally expect to find a decreased

cardiac output?

A) Hyperthyroidism

B) Beriberi

C) A-V fistula

D) Increased muscle mass

E) Hypothyroidism

85. Which of the following sets of changes would tend to increase

coronary blood flow?

Coronary Arteriole

Resistance

Cardiac Adenosine

Concentration

Coronary Vascular

Conductance

Cardiac

Workload

A)↑

↑ ↑ ↓

B)↑

↓ ↑ ↓

C)↑

↓ ↓ ↓

D)↑

↑ ↓ ↓

E)↓

↓ ↓ ↑

F)↓

↑ ↓ ↑

G)↓

↑ ↑ ↑

H)↓

↓ ↑ ↑

86. What will usually increase the plateau level of the cardiac output

curve?

A) Myocarditis

B) Severe cardiac tamponade

C) Decreased parasympathetic stimulation of the heart

D) Myocardial infarction

E) Mitral stenosis

87. If a person has been exercising for 1 hour, which organ will have

the smallest decrease in blood flow?

A) Brain

B) Intestines

C) Kidneys

D) Nonexercising skeletal muscle

E) Pancreas

88. A 35-year-old man has been diagnosed with a vitamin B1

deficiency. Oxygen consumption in this man is 400 ml/min. In

addition, pulmonary vein oxygen concentration is 200 ml/l of

blood, and pulmonary artery oxygen concentration is 150 ml/l of

blood. What is the cardiac output (l/min) in this man?

A) 4.0

B) 5.0

C) 6.0

D) 7.0

E) 8.0

89. Which vasoactive agent is usually the most important controller of

coronary blood flow?

A) Adenosine

B) Bradykinin

C) Prostaglandins

D) Carbon dioxide

E) Potassium ions

90. What will elevate the plateau of the cardiac output curve?

A) Surgically opening the thoracic cage

B) Connecting a patient to a mechanical ventilator

C) Cardiac tamponade

D) Increasing parasympathetic stimulation of the heart

E) Increasing sympathetic stimulation of the heart

91. The most likely cause of cardiac pain in acute ischemic coronary

disease is an increase in the extracellular concentration of the

following:

A) Adenosine

B) Potassium

C) Nitric oxide

D) ATP

E) Lactic acid

92. Which condition normally causes arteriolar vasodilation during

exercise?

A) Decreased plasma potassium ion concentration

B) Increased histamine release

C) Decreased plasma nitric oxide concentration

D) Increased plasma adenosine concentration

E) Decreased plasma osmolality

93. At the onset of exercise, the mass sympathetic nervous system

strongly discharges. What would you expect to occur?

A) Increased sympathetic impulses to the heart

B) Decreased coronary blood flow

C) Decreased cerebral blood flow

D) Reverse stress relaxation

E) Venous dilation

94. A sudden occlusion that occurs in larger coronary arteries causes

an increase in the following:

A) Dilation of small anastomoses in cardiac tissue

B) Increase collateral blood flow

C) Increase production of adenosine

D) All of the above

E) Only A and C

95. A 70-year-old man with a weight of 100 kg (220 lb) and a blood

pressure of 160/90 mm Hg has been told by his doctor that he has

angina caused by myocardial ischemia. Which treatment would be

beneficial to this man?

A) Increased dietary calcium

B) Isometric exercise

C) A beta-1 receptor stimulator

D) Angiotensin II infusion

E) Nitroglycerin

96. Which event normally occurs during exercise?

A) Arteriolar dilation in exercising muscle

B) Decreased sympathetic output

C) Venoconstriction

D) Decreased release of norepinephrine by the adrenals

E) A and C

97. Which of the following is (are) responsible for the increase in

stroke volume in response to increased venous return?

A) Stretch of right atrium initiates a nervous reflex called the

Bainbridge reflex

B) Stretch of the sinus node in the wall of the right atrium has a

direct effect on the rhythmicity of the node to increase the

heart rate

C) Frank-Starling law of the heart

D) All of the above

E) A and C

98. A 60-year-old man sustained an ischemia-induced myocardial

infarction and died from ventricular fibrillation. In this patient,

what factor was most likely to increase the tendency of the heart to

fibrillate after the infarction?

A) Low potassium concentration in the heart extracellular fluid

B) A decrease in ventricular diameter

C) Increased sympathetic stimulation of the heart

D) Low adenosine concentration

E) Decreased parasympathetic stimulation of the heart

99. A 60-year-old man has been told by his doctor that he has angina

caused by myocardial ischemia. Which treatment would be

beneficial to this man?

A) Angiotensin-converting enzyme inhibition

B) Isometric exercise

C) Chelation therapy such as ethylenediamine tetraacetic acid

(EDTA)

D) Beta receptor stimulation

E) Increased dietary calcium

100. What is one of the major causes of death after myocardial

infarction?

A) Increased cardiac output

B) A decrease in pulmonary interstitial volume

C) Fibrillation of the heart

D) Increased cardiac contractility

101. Which statement about the results of sympathetic stimulation is

most accurate?

A) Epicardial flow increases

B) Venous resistance decreases

C) Arteriolar resistance decreases

D) Heart rate decreases

E) Venous reservoirs constrict

102. What is normally associated with the chronic stages of

compensated heart failure? Assume the patient is resting.

A) Dyspnea

B) Decreased right atrial pressure

C) Decreased heart rate

D) Sweating

E) Increased mean systemic filling pressure

103. What normally occurs in a person with unilateral left heart

failure?

A) Decreased pulmonary artery pressure

B) Decreased left atrial pressure

C) Decreased right atrial pressure

D) Edema of feet

E) Increased mean pulmonary filling pressure

104. What normally causes renal sodium retention during

compensated heart failure?

A) Increased formation of angiotensin II

g

B) Increased release of atrial natriuretic factor

C) Sympathetic vasodilation of the afferent arterioles

D) Increased glomerular filtration rate

E) Increased formation of antidiuretic hormone (ADH)

105. Which intervention would normally be beneficial to a patient with

acute pulmonary edema?

A) Infuse a vasoconstrictor drug

B) Infuse a balanced electrolyte solution

C) Administer furosemide

D) Administer a bronchoconstrictor

E) Infuse whole blood

106. A 60-year-old man had a heart aack 2 days ago, and his blood

pressure has continued to decrease. He is now in cardiogenic

shock. Which therapy would be most beneficial?

A) Placing tourniquets on all four limbs

B) Administering a sympathetic inhibitor

C) Administering furosemide

D) Administering a blood volume expander

E) Increasing dietary sodium intake

107. If a 21-year-old male patient has a cardiac reserve of 300% and a

maximum cardiac output of 16 l/min, what is his resting cardiac

output?

A) 3 l/min

B) 4 l/min

C) 5.33 l/min

D) 6 l/min

E) 8 l/min

108. Which of the following occurs during heart failure and causes an

increase in renal sodium excretion?

A) Increased aldosterone release

B) Increased atrial natriuretic factor release

C) Decreased glomerular filtration rate

D) Increased angiotensin II release

E) Decreased mean arterial pressure

109. Which intervention would be appropriate therapy for a patient in

cardiogenic shock?

A) Placing tourniquets on the four limbs

B) Withdrawing a moderate amount of blood from the patient

C) Administering furosemide

g

D) Infusing a vasoconstrictor drug

110. Which condition normally accompanies acute unilateral right

heart failure?

A) Increased right atrial pressure

B) Increased left atrial pressure

C) Increased urinary output

D) Increased cardiac output

E) Increased arterial pressure

111. What is normally associated with the chronic stages of

compensated heart failure? Assume the patient is resting.

A) Decreased mean systemic filling pressure

B) Increased right atrial pressure

C) Increased heart rate

D) Sweating

E) Dyspnea

112. Patients with pulmonary edema often have dyspnea because of

accumulation of fluid in the lungs. Which of the following would

normally be the most beneficial for a patient with acute pulmonary

edema?

A) Infusing furosemide

B) Infusing dobutamine

C) Infusing saline solution

D) Infusing norepinephrine

E) Infusing whole blood

113. Which of the following is associated with compensated heart

failure?

A) Increased cardiac output

B) Increased blood volume

C) Decreased mean systemic filling pressure

D) Normal right atrial pressure

114. Which condition is normally associated with an increase in mean

systemic filling pressure?

A) Decreased blood volume

B) Congestive heart failure

C) Sympathetic inhibition

D) Venous dilation

115. Which condition normally occurs during the early stages of

compensated heart failure?

A) Increased right atrial pressure

g p

B) Normal heart rate

C) Decreased angiotensin II release

D) Decreased aldosterone release

E) Increased urinary output of sodium and water

116. What often occurs during decompensated heart failure?

A) Hypertension

B) Increased mean pulmonary filling pressure

C) Decreased pulmonary capillary pressure

D) Increased cardiac output

E) Increased norepinephrine in the endings of the cardiac

sympathetic nerves

117. Which of the following often occurs in decompensated heart

failure?

A) Increased renal loss of sodium and water

B) Decreased mean systemic filling pressure

C) Increased norepinephrine in cardiac sympathetic receptors

D) Orthopnea

E) Weight loss

118. An 80-year-old man at a local hospital was diagnosed with a heart

murmur. A chest radiograph showed an enlarged heart but no

edema fluid in the lungs. The mean QRS axis of his ECG was 170

degrees. His pulmonary wedge pressure was normal. What is the

diagnosis?

A) Mitral stenosis

B) Aortic stenosis

C) Pulmonary valve stenosis

D) Tricuspid stenosis

E) Mitral regurgitation

119. The fourth heart sound is associated with which mechanism?

A) In-rushing of blood into the ventricles from atrial contraction

B) Closing of the A-V valves

C) Closing of the pulmonary valve

D) Opening of the A-V valves

E) In-rushing of blood into the ventricles in the early to middle

part of diastole

120. A 40-year-old woman has been diagnosed with a heart murmur.

A “ blowing” murmur of relatively high pitch is heard maximally

over the left ventricle. The chest radiograph shows an enlarged

heart. Arterial pressure in the aorta is 140/40 mm Hg. What is the

diagnosis?

A) Aortic valve stenosis

B) Aortic valve regurgitation

C) Pulmonary valve stenosis

D) Mitral valve stenosis

E) Tricuspid valve regurgitation

121. In which disorder will left ventricular hypertrophy normally

occur?

A) Pulmonary valve regurgitation

B) Tricuspid regurgitation

C) Mitral stenosis

D) Tricuspid stenosis

E) Aortic stenosis

122. Which heart murmur is heard during systole?

A) Aortic valve regurgitation

B) Pulmonary valve regurgitation

C) Tricuspid valve stenosis

D) Mitral valve stenosis

E) Patent ductus arteriosus

123. An increase in left atrial pressure is most likely to occur in which

heart murmur?

A) Tricuspid stenosis

B) Pulmonary valve regurgitation

C) Aortic stenosis

D) Tricuspid regurgitation

E) Pulmonary valve stenosis

124. A 50-year-old woman at a local hospital has been diagnosed with

a heart murmur. A murmur of relatively low pitch is heard

maximally over the second intercostal space to the right of the

sternum. The chest radiograph shows an enlarged heart. The mean

QRS axis of the ECG is −45 degrees. What is the diagnosis?

A) Mitral valve stenosis

B) Aortic valve stenosis

C) Pulmonary valve stenosis

D) Tricuspid valve stenosis

E) Tricuspid valve regurgitation

125. A 40-year-old woman has been diagnosed with a heart murmur of

relatively high pitch heard maximally in the second intercostal

y g p y

space to the left of the sternum. The mean QRS axis of his ECG is

150 degrees and the chest radiographs show an enlarged heart. The

arterial blood oxygen content is normal. What is the likely

diagnosis?

A) Aortic stenosis

B) Aortic regurgitation

C) Pulmonary valve regurgitation

D) Mitral stenosis

E) Tricuspid stenosis

126. In which condition will right ventricular hypertrophy normally

occur?

A) Tetralogy of Fallot

B) Mild aortic stenosis

C) Mild aortic insufficiency

D) Mitral stenosis

E) Tricuspid stenosis

127. Which heart murmur is only heard during diastole?

A) Patent ductus arteriosus

B) Aortic stenosis

C) Tricuspid valve regurgitation

D) Interventricular septal defect

E) Mitral stenosis

128. A person with which condition is most likely to have low arterial

oxygen content?

A) Tetralogy of Fallot

B) Pulmonary artery stenosis

C) Tricuspid insufficiency

D) Patent ductus arteriosus

E) Tricuspid stenosis

129. Which of the following is associated with the first heart sound?

A) Inrushing of blood into the ventricles as a result of atrial

contraction

B) Closing of the A-V valves

C) Closing of the pulmonary valve

D) Opening of the A-V valves

E) Inrushing of blood into the ventricles in the early to middle

part of diastole

130. A 2-year-old girl had an echocardiogram. The results indicated a

thickened right ventricle. Other data indicated that the patient had

g p

severely decreased arterial oxygen content and equal systolic

pressures in both cardiac ventricles. What condition is present?

A) Interventricular septal defect

B) Tetralogy of Fallot

C) Pulmonary valve stenosis

D) Pulmonary valve regurgitation

E) Patent ductus arteriosus

131. Which heart murmur is only heard during diastole?

A) Patent ductus arteriosus

B) Mitral regurgitation

C) Tricuspid valve stenosis

D) Interventricular septal defect

E) Aortic stenosis

132. Which mechanism is associated with the third heart sound?

A) Inrushing of blood into the ventricles as a result of atrial

contraction

B) Closing of the A-V valves

C) Closing of the pulmonary valve

D) Opening of the A-V valves

E) Inrushing of blood into the ventricles in the early to middle

part of diastole

133. Which condition often occurs in a person with progressive

hemorrhagic shock?

A) Increased capillary permeability

B) Stress relaxation of veins

C) Tissue alkalosis

D) Increased urine output

E) Increased mean systemic filling pressure

134. In which condition will administration of a sympathomimetic

drug be the therapy of choice to prevent shock?

A) Spinal cord injury

B) Shock due to excessive vomiting

C) Hemorrhagic shock

D) Shock caused by excess diuretics

135. The blood pressure of a 60-year-old man decreased to 55/35 mm

Hg during induction of anesthesia. His ECG still shows a normal

sinus rhythm. What initial therapy do you recommend?

A) Infusion of packed red blood cells

B) Infusion of plasma

p

C) Infusion of a balanced electrolyte solution

D) Infusion of a sympathomimetic drug

E) Administration of a glucocorticoid

136. A 65-year-old man enters a local emergency department a few

minutes after receiving an influenza inoculation. He has pallor,

tachycardia, arterial pressure of 80/50, and trouble walking. What

therapy do you recommend to prevent shock?

A) Infusion of blood

B) Administration of an antihistamine

C) Infusion of a balanced electrolyte solution such as saline

D) Infusion of a sympathomimetic drug

E) Administration of tissue plasminogen activator

137. Which condition often occurs in compensated hemorrhagic

shock? Assume systolic pressure is 48 mm Hg.

A) Decreased heart rate

B) Stress relaxation of veins

C) Decreased ADH release

D) Decreased absorption of interstitial fluid through the

capillaries

E) Central nervous system (CNS) ischemic response

138. If a patient undergoing spinal anesthesia experiences a large

decrease in arterial pressure and goes into shock, what would be

the therapy of choice?

A) Plasma infusion

B) Blood infusion

C) Saline solution infusion

D) Glucocorticoid infusion

E) Infusion of a sympathomimetic drug

139. A 25-year-old man who has been in a motorcycle wreck enters the

emergency department. His clothes are very bloody, and his

arterial pressure is decreased to 70/40 mm Hg. His heart rate is 120

beats/min, and his respiratory rate is 30/min. Which therapy would

the physician recommend?

A) Infusion of blood

B) Infusion of plasma

C) Infusion of a balanced electrolyte solution

D) Infusion of a sympathomimetic drug

E) Administration of a glucocorticoid

140. In which type of shock does cardiac output often increase?

yp p

A) Hemorrhagic shock

B) Anaphylactic shock

C) Septic shock

D) Neurogenic shock

141. A 20-year-old man who has been hemorrhaging as a result of a

gunshot wound enters a local emergency department. He has pale

skin, tachycardia, an arterial pressure of 60/40 mm Hg, and trouble

walking. Unfortunately, the blood bank is out of whole blood.

Which therapy would the physician recommend to prevent shock?

A) Administration of a glucocorticoid

B) Administration of an antihistamine

C) Infusion of a balanced electrolyte solution

D) Infusion of a sympathomimetic drug

E) Infusion of plasma

142. A 10-year-old girl in the hospital had an intestinal obstruction,

and her arterial pressure decreased to 70/40 mm Hg. Her heart rate

is 120 beats/min, and her respiratory rate is 30/min. Which therapy

would the physician recommend?

A) Infusion of blood

B) Infusion of plasma

C) Infusion of a balanced electrolyte solution

D) Infusion of a sympathomimetic drug

E) Administration of a glucocorticoid

143. What often occurs during progressive shock?

A) Patchy areas of necrosis in the liver

B) Decreased tendency for blood to clot

C) Increased glucose metabolism

D) Decreased release of hydrolases by lysosomes

E) Decreased capillary permeability

144. Release of which substance causes vasodilation and increased

capillary permeability during anaphylactic shock?

A) Histamine

B) Bradykinin

C) Nitric oxide

D) Atrial natriuretic factor

E) Adenosine

145. A 36-year-old female has a resting cardiac output (CO) of 4.8

l/min and after maximum exercise increased to 19.2 l/min. What is

(approximately) her cardiac reserve?

pp y

A) 400%

B) 300%

C) 500%

D) Cannot be estimated without mean arterial pressure values

E) Cannot be estimated without total peripheral resistance

values

146. A 58-year-old patient with a history of atherosclerosis and

hypertension suffers a heart aack. What are acute events that take

place immediately (0–30 seconds) after heart damage?

A) Increased cardiac output

B) Blood accumulation in the aorta

C) Sympathetic activation

D) Parasympathetic activation

E) Inhibition of angiotensin II

147. What is incorrect about cardiac failure?

A) Moderate fluid retention is beneficial

B) Cardiac reserve is decreased only when ejection fraction is

less than 30%

C) Cardiac failure may develop with high or low cardiac output

D) Cardiac recovery is possible but cardiac reserve is always

decreased

E) A low cardiac output tends to decrease urinary output

148. An important reason why moderate fluid retention in low-output

heart failure is beneficial is:

A) Preserves isovolumetric contraction

B) Increases afterload

C) Improves preload

D) Reduces aortic pressure

E) Decreases peripheral edema

149. In decompensated heart failure, the failure of CO to rise enough will

result in:

A) Progressive fluid retention, increased mean filling pressure,

and increased right atrial pressure

B) Progressive parasympathetic activation, decreased

aldosterone, increased heart rate.

C) Moderate fluid retention, increased mean filling pressure,

decreased venous return

D) Vasoconstriction, bronchospasm, and decreased right atrial

pressure

p

E) Stable fluid retention, increased mean filling pressure, and

increased right atrial pressure

150. A 67-year-old man has an ejection fraction of 0.32, no cyanosis, a

history of dilated cardiomyopathy and heart failure, and a systolic

murmur. What is your most likely diagnosis?

A) Mitral stenosis

B) Tetralogy of Fallot

C) Mitral regurgitation

D) Patent ductus arteriosus

E) Tricuspid stenosis

151. The murmur in mitral stenosis is due to:

A) Increased pulmonary pressures

B) Narrowed outflow tract of the left ventricle

C) Backflow from atria to the pulmonary vessels

D) Narrowed mitral valve opening

E) A and D

152. What is correct about interpretation of left ventricular pressurevolume loops in valve disease?

A) Aortic stenosis shows a taller P-V loop with reduced preload.

B) Isovolumetric systolic period is lost in aortic regurgitation but

preserved in mitral regurgitation

C) Aortic stenosis and regurgitation show a significantly

increased afterload.

D) Mitral stenosis and regurgitation show a significantly

increased afterload.

E) Isovolumetric diastolic period is lost in tricuspid stenosis.

153. A 6-month-old patient has a chest X-ray showing enlargement of

the heart and blood work with low Po2

. Which situation best

explains his condition?

A) Patent ductus arteriosus

B) A right-to-left shunt

C) A left-to-right shunt

D) Congenital tricuspid stenosis

E) Interatrial septal defect

154. Which of the following is correct about hemorrhagic shock?

A) Deterioration of the heart is probably the most important

factor in progression of shock

B) Deterioration of the liver is probably the most important

factor in progression of shock

C) Autoregulation in the brain reverses cellular deterioration in

irreversible shock

D) Autoregulation in the heart reverses cellular deterioration in

irreversible shock

E) Autoregulation in the kidneys reverses cellular deterioration

in irreversible shock

155. A 48-year-old male suffers a massive heart aack that deteriorates

over 70% of his left ventricle (LV) (EKG shows ST elevation from

V1 to V6, in lead I, and in aVL). The blood pressure is 82/57 mm

Hg, heart rate is 135 beats/min, pulse is weak and the patient

displays generalized signs of hypoperfusion (lethargic, pale, sweat,

cold skin). The diagnosis of the heart aack was done at his home 7

hours ago. The patient was admied 55 minutes ago and has been

receiving IV fluids, oxygen, and sympathomimetics He showed a

brief improvement in blood pressure and cardiac dynamics but

then continued to deteriorate with no further response to

treatment. Which of the following statements is most likely to be

correct?

A) The delay between diagnosis and admiance at hospital

complicated the patients’ hemorrhagic shock with a

progressive decrease in capillary permeability

B) It is possible that fluids may have been insufficient and

increased administration of blood + fluids may reverse cellular

deterioration

C) The patient is most likely at the irreversible stage of shock

D) The diminished delivery of oxygen to the tissues leads to

generalized tissue alkalosis and cellular deterioration

E) Since the cause of shock is from cardiac origin, no toxins are

released or accumulated and cellular deterioration may not

develop.

156. The patient from the previous question, after 2 hours of

therapeutic interventions, develops ventricular fibrillation. Which

of the following will represent the most significant change in his

left ventricular cardiovascular dynamics?

A) A significant increase in preload

B) A decrease in afterload with preserve preload

C) A circulatory arrest


















guyton and hall physiology review
guyton and hall physiology
guyton and hall physiology review mcq
guyton and hall physiology short notes
guyton and hall physiology review latest edition
guyton and hall physiology review questions pdf
guyton and hall physiology mcqs
guyton and hall physiology review 4th edition
guyton and hall physiology lectures

guyton and hall physiology ppt
guyton and hall physiology anki
guyton and hall physiology anki deck
guyton and hall physiology audiobook
guyton and hall textbook of medical physiology apa citation
guyton and hall medical physiology 13th edition amazon
guyton and hall textbook of medical physiology south asian edition pdf download
guyton and hall textbook of medical physiology 13th edition amazon
guyton and hall textbook of medical physiology 14th edition amazon
guyton and hall physiology south asia edition pdf download
guyton and hall physiology south asia edition pdf
guyton and hall physiology book price
guyton and hall physiology book price in pakistan
guyton and hall physiology book
guyton and hall medical physiology book
guyton hall physiology question book
guyton and hall textbook of medical physiology e-book
physiology by guyton and hall
medical physiology by guyton and hall
guyton and hall textbook of medical physiology book buy
guyton and hall medical physiology review book
guyton and hall physiology chapters
guyton and hall physiology contents
guyton and hall physiology chapter 1
guyton and hall physiology citation
guyton and hall physiology table of contents
guyton and hall textbook of medical physiology citation
guyton and hall textbook of medical physiology chapters
guyton and hall textbook of medical physiology cite
guyton and hall textbook of medical physiology citation apa
guyton and hall textbook of medical physiology content
guyton and hall textbook of medical physiology deutsch
guyton and hall textbook of medical physiology 14th ed
guyton and hall textbook of medical physiology 12th ed citation
guyton and hall medical physiology ebook
guyton and hall human physiology
guyton and hall physiology kit
guyton and hall physiology lecture ppt
guyton and hall physiology latest
guyton and hall textbook of medical physiology latest international edition
guyton and hall physiology medical
guyton and hall medical physiology 14th
guyton and hall medical physiology notes
guyton and hall medical physiology review
guyton and hall textbook of medical physiology 14th edition citation
guyton and hall textbook of medical physiology 12th ed
guyton and hall physiology notes
guyton and hall textbook of medical physiology online
guyton and hall of medical physiology
guyton and hall textbook of medical physiology 14th
guyton and hall textbook of medical physiology 12th edition citation
guyton and hall physiology price
guyton and hall textbook of medical physiology price
guyton and hall textbook of medical physiology ppt
guyton and hall physiology questions
guyton and hall physiology quizlet
guyton and hall physiology review questions
guyton and hall textbook of medical physiology quizlet
guyton and hall textbook of medical physiology quiz
guyton and hall physiology review questions 4th edition pdf
guyton and hall physiology review questions 4th edition
guyton and hall textbook of medical physiology quora
guyton and hall physiology review 3rd edition
guyton and hall physiology review 2nd edition
guyton and hall physiology south asia edition
guyton and hall physiology summary
guyton and hall textbook of medical physiology summary
guyton and hall physiology third south asia edition pdf download
guyton and hall textbook of medical physiology south asian edition
guyton and hall textbook of medical physiology 2nd south asia edition pdf
guyton and hall physiology textbook
guyton and hall medical physiology textbook
guyton and hall physiology videos
guyton and hall textbook of medical physiology videos
guyton and hall textbook of medical physiology tiếng việt
guyton and hall physiology x ray
guyton and hall physiology xr
guyton and hall physiology youtube
guyton and hall physiology zotero
guyton and hall physiology zone
guyton and hall physiology zuid korea
can guyton and hall physiology exam
can guyton and hall physiology edition
can guyton and hall physiology essay
can guyton and hall physiology enzymes
can guyton and hall physiology journal
can guyton and hall physiology jobs
can guyton and hall physiology journal impact factor
can guyton and hall physiology kit
can guyton and hall physiology keys
can guyton and hall physiology of arts
can guyton and hall physiology of medicine
guyton and hall renal physiology
guyton and hall respiratory physiology
can guyton and hall physiology wikipedia
can guyton and hall physiology wiki
can guyton and hall physiology workshop
can guyton and hall physiology x ray
can guyton and hall physiology xr
can guyton and hall physiology youtube
can guyton and hall physiology yacht
can guyton and hall physiology zone
can guyton and hall physiology zurich
can guyton and hall physiology zotero
how guyton and hall physiology essay
how guyton and hall physiology exam
how guyton and hall physiology edition
how guyton and hall physiology endnote
how guyton and hall physiology journal
how guyton and hall physiology journal impact factor
how guyton and hall physiology jobs
how guyton and hall physiology kit
how guyton and hall physiology key
how guyton and hall physiology keys
how guyton and hall physiology of arts
how guyton and hall physiology of medicine
how guyton and hall physiology wikipedia
how guyton and hall physiology wiki
how guyton and hall physiology workshop
how guyton and hall physiology x ray
how guyton and hall physiology youtube
how guyton and hall physiology yacht
how guyton and hall physiology zone
how guyton and hall physiology zurich
how guyton and hall physiology zotero
how guyton and hall physiology zones
introduction to physiology
human physiology
الفيزيولوجيا الطبية
محاضرات physiology
which guyton and hall physiology edition
which guyton and hall physiology exam
which guyton and hall physiology essay
which guyton and hall physiology journal
which guyton and hall physiology jobs
which guyton and hall physiology journal impact factor
which guyton and hall physiology kit
which guyton and hall physiology key
which guyton and hall physiology keys
which guyton and hall physiology of medicine
which guyton and hall physiology of arts

F, Female; M, Male.

Comments

Search This Blog

Archive

Show more

Popular posts from this blog

TRIPASS XR تري باس

CELEPHI 200 MG, Gélule

ZENOXIA 15 MG, Comprimé

VOXCIB 200 MG, Gélule

Kana Brax Laberax

فومي كايند

بعض الادويه نجد رموز عليها مثل IR ، MR, XR, CR, SR , DS ماذا تعني هذه الرموز

NIFLURIL 700 MG, Suppositoire adulte

Antifongiques مضادات الفطريات

Popular posts from this blog

علاقة البيبي بالفراولة بالالفا فيتو بروتين

التغيرات الخمس التي تحدث للجسم عند المشي

إحصائيات سنة 2020 | تعداد سكَان دول إفريقيا تنازليا :

ما هو الليمونير للأسنان ؟

ACUPAN 20 MG, Solution injectable

CELEPHI 200 MG, Gélule

الام الظهر

VOXCIB 200 MG, Gélule

ميبستان

Popular posts from this blog

TRIPASS XR تري باس

CELEPHI 200 MG, Gélule

Popular posts from this blog

TRIPASS XR تري باس

CELEPHI 200 MG, Gélule

ZENOXIA 15 MG, Comprimé

VOXCIB 200 MG, Gélule

Kana Brax Laberax

فومي كايند

بعض الادويه نجد رموز عليها مثل IR ، MR, XR, CR, SR , DS ماذا تعني هذه الرموز

NIFLURIL 700 MG, Suppositoire adulte

Antifongiques مضادات الفطريات

Popular posts from this blog

Kana Brax Laberax

TRIPASS XR تري باس

PARANTAL 100 MG, Suppositoire بارانتال 100 مجم تحاميل

الكبد الدهني Fatty Liver

الم اسفل الظهر (الحاد) الذي يظهر بشكل مفاجئ bal-agrisi

SEDALGIC 37.5 MG / 325 MG, Comprimé pelliculé [P] سيدالجيك 37.5 مجم / 325 مجم ، قرص مغلف [P]

نمـو الدمـاغ والتطـور العقـلي لـدى الطفـل

CELEPHI 200 MG, Gélule

أخطر أنواع المخدرات فى العالم و الشرق الاوسط

Archive

Show more